Jamais rien ne tomba dans un trou noir ? - Page 2
Discussion fermée
Page 2 sur 5 PremièrePremière 2 DernièreDernière
Affichage des résultats 31 à 60 sur 121

Jamais rien ne tomba dans un trou noir ?



  1. #31
    Avatar10

    Re : Jamais rien ne tomba dans un trou noir ?


    ------

    Citation Envoyé par bernarddo Voir le message
    Il faudrait un peu de rigueur dans les concepts (1)
    C'est vrai...

    1 Il n'y a pas de géométrie de Schwarzschild, mais une métrique qui permet de décrire l'espace-temps
    Alors en toute rigueur, une métrique, cela permet de décrire la géométrie de l'ET...et y poser plein de trucs...
    Un petit cours très élémentaire: http://mespages.univ-brest.fr/~scott..._lecture_6.pdf
    Vous auriez eu de la curiosité, vous auriez trouvé en qq sec plusieurs cours traitant de ça...
    Quand à la logique, il me semble que vous abordez des choses pour lesquelles vous n'êtes pas suffisamment formé (cela se voit à vos questions, et la façon dont vous les posées, ainsi que leurs pertinence), donc revenez en arrière, c'est ce qu'il y a de plus logique à faire.
    Pour ma part, je ne pense pas qu'un forum soit fait pour prendre un cours complet, donc je m'arrête là avec vous.
    Bonne continuation.

    -----
    Dernière modification par Avatar10 ; 12/07/2021 à 22h47.

  2. #32
    mach3
    Modérateur

    Re : Jamais rien ne tomba dans un trou noir ?

    Citation Envoyé par bernarddo Voir le message
    La résolution de l'équation de champ (solution extérieure de Schwarzschild) ne doit pas être comprise comme une narration d'un évènement, par exemple d'une formation de trou noir.

    Toute référence à ce type d'évènement est donc hors de propos.

    C'est une expérience de pensée, partant d'une hypothèse physique mathématiquement singulière (existence d'une masse ponctuelle finie, de dimension aussi petite que l'on veut, ce qui constitue une singularité physique, en correspondance logique à la singularité mathématique), qui prétend simplement en déduire la description de l'espace-temps correspondant.

    Dans sa résolution mathématique, que vous trouverez à arXiv:gr-qc/0406090

    "Reinstating Schwarzschild's original manifold and its singularity" ,

    vous découvrirez que sa résolution originale donne une solution différente de celle qui porte son nom, et que celà est dû à son exigence de maintenir une condition de continuité, qu'il a résolue en faisant coïncider une discontinute mathématique aparue dans lecalcul avec l'origine du système d'axe, où se situait la singularité physique.

    D'où l'existence d'un ET limité au rayon de Schwarzschild autour de cette masse, lequel constitue une frontière temporelle et spatiale
    de cet espace-temps particulier et non physique.
    Les annexes de arXiv:gr-qc/0406090 sont très intéressantes puisque ce sont les traductions des articles de Schwarzschild et de Hilbert que vous avez l'habitude d'opposer (je ne commente pas le reste de l'article qui semble un torchon au premier abord vu qu'il est orthogonal au consensus habituel et que je n'ai absolument pas le temps en ce moment d'étudier ce genre de trucs, déjà que je n'ai pas vraiment le temps pour vous répondre mais que ça me démangeait et que je n'ai pas le temps de faire des trucs qui m'intéressent vraiment en ce moment, c'est pas pour le gaspiller à étudier des daubes)

    On voit en A.14, que Schwarzschild abouti à l'expression de suivante pour la métrique, mais j'y remplace sa notation par la notation , à dessein :

    , avec , et si on remonte plus loin, , , et étant des coordonnées "rectangulaires", autre nom pour coordonnées cartésiennes. est donc positif et supérieur à .

    On reconnait la métrique de la sphère, , donc le coefficient qui précède, , n'est autre que ce qu'on appelle le rayon aréal, , au carré.

    L'expression décrit donc la géométrie d'un ouvert d'une variété, ouvert sur lequel on a défini 4 champs scalaires, , , et qui varient sur des intervalles bien définis (, , hérités des coordonnées sphériques, et ). La signature, nous montre que sur ces intervalles là, est de genre temps alors que , et sont de genre espace.

    De son côté, Hilbert abouti à l'expression suivante en B.45, mais j'y remplace sa notation par la notation , à dessein :



    n'est pas clairement défini, à part pour définir trois fonctions , et en début d'article (elles semblent correspondre aux , et de l'article de Schwarzschild, mais nous allons voir que ce n'est pas le cas), trois fonctions qui ne sont pas utilisées ensuite.

    Si on réarrange l'expression (et qu'on inverse la convention de signature), on a :



    C'est donc la même expression que A.14, le rayon aréal est simplement noté chez Schwarzschild, mais chez Hilbert.

    La différence entre Schwarzschild et Hilbert n'est pas l'expression de la métrique, vu que c'est la même (et ne venez pas dire le contraire, il s'agit d'une évidence factuelle, tout le monde peut le vérifier). C'est la définition des variables au départ qui est différente. Schwarzschild part avec des coordonnées cartésiennes , et , génère ensuite des coordonnées sphériques , et d'une façon "naïve" telles que , tout ça pour être obligé d'introduire une nouvelle coordonnée radiale , fonction de la première qui se trouve être le rayon aréal . Hilbert part de coordonnées , et qui ne sont pas , et , car en générant ses coordonnées sphériques , et , est déjà le rayon aréal .

    Ce sera tout pour la mise au point d'aujourd'hui. Il y aurait encore beaucoup de choses à dire, par exemple sur que sont et , ou encore sur le fait que l'ouvert considéré (avec 4 champs scalaire tels que , , et ) ne peut pas être la variété espace-temps complète vu que certaines de ces géodésiques peuvent être prolongées.

    m@ch3
    Dernière modification par mach3 ; 13/07/2021 à 11h55. Motif: boulette
    Never feed the troll after midnight!

  3. #33
    bernarddo

    Re : Jamais rien ne tomba dans un trou noir ?

    Citation Envoyé par Avatar10 Voir le message

    Alors en toute rigueur, une métrique, cela permet de décrire la géométrie de l'ET...et y poser plein de trucs...
    Donc Schwarzschild avait la simple ambition de décrire la géométrie de l'ET, dans des cas d'ailleurs très simplifiés, on est donc d'accord là dessus.

    Citation Envoyé par Avatar10 Voir le message
    Un petit cours très élémentaire: http://mespages.univ-brest.fr/~scott..._lecture_6.pdf
    Vous auriez eu de la curiosité, vous auriez trouvé en qq sec plusieurs cours traitant de ça...
    Quand à la logique, il me semble que vous abordez des choses pour lesquelles vous n'êtes pas suffisamment formé...
    Bonne continuation.
    Je dois enfin reconnaître, et te remercier, du fait que ton dernier post me donne l'occasion de confesser publiquement une insuffisance de formation, particulièrement adéquate quant à ce qui nous oppose. Il se trouve que je suis dans l'incapacité de comprendre certaines choses dans le petit cours très élémentaire dont tu donnes gracieusement les coordonnées.

    Outre le fait que les planches sont d'une lecture très pénible, et qui sent à plein nez la traduction automatique, j'avoue, le rouge au front, que je ne pige rien à la planche 11.1 (ou 11 bis).

    Intervalle.JPG

    Pour quelqu'un comme moi qui en était resté au théorème de Pythagore, le déplacement élémentaire dl dans notre espace réel, racine d'un carré, était forcément un réel.
    Le fait qu'il s'agisse d'un déplacement imaginaire, puisqu'il faut recourir à la notation imaginaire pour respecter l'orthodoxie mathématique, représente quelque chose qui dépasse mon niveau, moi qui n'avais jamais pensé que notre univers était imaginaire.


    Heureusement, le fait que Schwarzschild partage ma confusion, en considérant aussi les déplacements comme réels, est suffisant pour me rassurer!
    (suivant autre extrait de sa démonstration).


    signature.JPG

  4. #34
    Deedee81
    Modérateur

    Re : Jamais rien ne tomba dans un trou noir ?

    Salut,

    Citation Envoyé par bernarddo Voir le message
    Heureusement, le fait que Schwarzschild partage ma confusion, en considérant aussi les déplacements comme réels, est suffisant pour me rassurer!
    (suivant autre extrait de sa démonstration).
    Je ne comprend pas. Il ne parle pas de déplacement réel dans cet extrait, on a juste des calculs avec des coordonnées.
    "Il ne suffit pas d'être persécuté pour être Galilée, encore faut-il avoir raison." (Gould)

  5. #35
    mach3
    Modérateur

    Re : Jamais rien ne tomba dans un trou noir ?

    Citation Envoyé par bernarddo Voir le message
    j'avoue, le rouge au front, que je ne pige rien à la planche 11.1 (ou 11 bis).
    C'est pourtant le B.A.BA en relativité. Si cela n'est pas compris, alors il est inutile de venir donner des leçons sur l'interprétation que l'on doit donner à la métrique de Schwarzschild. C'est une perte de temps pour tout le monde.

    Pour quelqu'un comme moi qui en était resté au théorème de Pythagore, le déplacement élémentaire dl dans notre espace réel, racine d'un carré, était forcément un réel.
    Dans une variété Riemannienne, le carré scalaire d'un vecteur non nul (du tangent), c'est à dire l'application du tenseur métrique deux fois sur ce même vecteur , donne bien toujours un réel positif (). Ainsi on peut prendre la racine carré pour connaitre la longueur du vecteur u (). Souvent (la métrique appliquée à deux vecteurs non spécifiés) est noté mais c'est une notation impropre et fortement critiquable bien que très usitée (tout comme de dire que ds ou dl sont des nombres et parler de leurs signes!!!). Si on écrit , on sous-entend qu'on a pris le carré scalaire d'un vecteur et qu'on a obtenu un réel positif.

    Dans une variété Lorentzienne, le carré scalaire d'un vecteur non nul (du tangent), c'est à dire l'application du tenseur métrique deux fois sur ce même vecteur peut donner un réel positif, négatif ou nul. C'est la particularité de ces variétés dont l'exemple le plus simple est l'espace-temps de Minkowski : les vecteurs ont un genre et ce genre est donné par le signe de leur carré scalaire. Dans la notation impropre décrite dans le paragraphe précédent, cela veut dire qu'on peut avoir, selon le vecteur dont on prend le carré scalaire , ou , mais n'est qu'une notation maladroite, ce n'est pas le carré d'un nombre qui serait noté et qui devrait donc être imaginaire si .
    En fonction du genre, il faut pour connaitre la longueur ou la durée (cela dépend du genre et de la convention de signe) du vecteur prendre la racine carré de soit son opposé afin d'avoir un réel positif sous le radical et donc une longueur ou une durée réelle.
    Personne ne parle de déplacement imaginaire et il n'y a pas de notation imaginaire ni d'univers imaginaire(*).

    Heureusement, le fait que Schwarzschild partage ma confusion, en considérant aussi les déplacements comme réels, est suffisant pour me rassurer!
    Il y a probablement des confusions chez Schwarzschild, à cause de la précocité de ses travaux, mais surement pas celle-là...

    m@ch3

    *: historiquement il y a eu une passe où on considérait un nombre imaginaire l=it (par exemple dans l'article de Hilbert) comme coordonnée temporelle, ce qui donnait des expression métrique avec des signes positifs devant tous les termes, mais cela s'est avéré ensuite d'aucun intérêt.
    Never feed the troll after midnight!

  6. #36
    Deedee81
    Modérateur

    Re : Jamais rien ne tomba dans un trou noir ?

    Citation Envoyé par mach3 Voir le message
    *: historiquement il y a eu une passe où on considérait un nombre imaginaire l=it (par exemple dans l'article de Hilbert) comme coordonnée temporelle, ce qui donnait des expression métrique avec des signes positifs devant tous les termes, mais cela s'est avéré ensuite d'aucun intérêt.
    Je trouve même ça source de confusion (et je l'ai déjà vu sur le forum)
    "Il ne suffit pas d'être persécuté pour être Galilée, encore faut-il avoir raison." (Gould)

  7. #37
    bernarddo

    Re : Jamais rien ne tomba dans un trou noir ?

    Bonjour,

    Je me limiterai ici à exposer les éléments les plus basiques (sur les plans de la physique et des mathématiques) pour expliquer ce qui m’amène à choisir la solution de Schwarzschild par rapport à celle qui lui a été attribués par la communauté. Commenter des communications de Schwarzschild ou d’Hilbert pour essayer de les éclairer me semble respecter la charte. Les dernières interventions des modérateurs, que je remercie, m’ont d’ailleurs beaucoup guidé dans cette entreprise.

    A Pourquoi la solution de Schwarzschild me paraît l’emporter :

    1 Elle possède une description mathématique adéquate, celle indiquée par mach3 : ( variété à 4 champs scalaires avec leurs intervalles de définition), que l’on l’écrive avec le marqueur d’espace R ou A (de même définition).

    variété.JPG

    2 Considérant les intervalles de variation , on peut l’interpréter comme un ET de symétrie sphérique, s’étendant jusqu’à devenir à l’infini un espace de Minkovski, et borné du côté de l’origine des coordonnées par un bord intérieur à la distance R (ou A, rayon aréal) de cette origine où se situe la masse ponctuelle.
    La signature de la métrique (+,-,-,-,) est la même qui décrit l’espace de Minkovski que l’on retrouve à distance infinie de la masse, ce qui traduit à la fois le respect de la continuité que Schwarzschild avait inclus dans son calcul, et sa revendication d’une symétrie sphérique, au contraire de la symétrie centrale revendiquée par Hilbert.
    Sch :

    SCh3.JPG

    Hilbert :

    Sch2.JPG

    3 Cerise sur le gâteau, un changement de variables astucieux permet de valider son interprétation topologique, en annulant toutes les singularités, celle du centre par l’absence de l’ET, et celle à r =Rs, apparaissant comme une singularité de coordonnées qui peut être surmontée par ce changement de variables, rendant la métrique parfaitement régulière.

    Métrique.JPG

    D’où la continuité des géodésiques de cet ET sans centre, et leur impossibilité d’entrer dans le rayon de Schwarzschild, vide d’espace, ce qui donne un démenti mathématique formel à l’argument de mach3 selon lequel l’ouvert décrivant la solution de Schwarschild ne peut être la solution complète car « certaines » géodésiques pourraient être prolongées.

    On trouve ce changement de coordonnées sur Futura, sur le fil « Unification mécanique quantique relativité générale, (dont je suis désolé que le titre n’ait rien à voir avec le sujet), posts 9 et 20.


    Tout s’explique clairement, sa description mathématique, sa visualisation spatiale et sa continuité revendiquées par son auteur, et l’existence d’un système de coordonnées qui permettent de « visualiser » sa topologie tout en réfutant l’argumentation factuelle la plus forte qui lui est opposée


    B Pourquoi je récuse la solution classique :

    Les raisons en sont plus générales, car elles me paraissent suffisantes, les spécialistes pourront détailler.

    Je suis encore d’accord avec mach3, quand il affirme que le choix de variables de départ joue un rôle.

    En effet, bien qu’elles paraissent équivalentes, et revendiquées choisies en référence à Schwarzschild, les coordonnées posées par Hilbert (ω1, ω2, ω3, ω4 =l), paraissent certes correspondre respectivement avec x, z, y et t., mais avec la différence essentielle que Hilbert pose l = it, la coordonnée temporelle serait donc devenue imaginaire.

    Nous nous trouvons alors dans un système de coordonnées hybride (réel/imaginaire), dans lequel les marqueurs d’espace sont réels, et celui de temps imaginaire. Une interrogation assez naturelle est alors de se demander s’il est bien raisonnable de s’appuyer sur une chimère pour décrire le réel. C’est en tout cas la mienne.

    D’où la réflexion que c’est précisément cette chimère qui entraîne naturellement d’avoir à considérer le, chimérique pour moi, interchange du temps et de l’espace, celui revendiqué par Aurélien Barrau dans sa conférence, simplement pour pouvoir faire exister dans ce référentiel baroque, l’impossible coexistence d’un temps propre réel avec des trajectoires réelles à la fois à l’extérieur et à l’intérieur du rayon de Schwarzschild

    Nota sur l’existence de variétés à ds2 positif , nul et négatif, l’argument académique que m’oppose mach3.

    Il n'est nullement question de le contester: simplement, je ferai le parallèle avec une propriété beaucoup plus triviale selon laquelle une équation polynomiale de degré n possède n solutions ou racines. C’est évidemment tout aussi vrai, il suffit simplement d’intégrer la possibilité de racines imaginaires.

  8. #38
    mach3
    Modérateur

    Re : Jamais rien ne tomba dans un trou noir ?

    Pourquoi la solution de Schwarzschild me paraît l’emporter
    mais l'emporter sur quoi ? Il n'y a qu'une seule et unique solution pour un espace-temps de symétrie sphérique, vide et asymptotiquement plat, une variété qui présente une géométrie dite "de Schwarzschild". On peut construire une infinité de système de coordonnées sur cette variété et il y a un système de coordonnées particulier, dit "de Schwarzschild", où l'expression de la métrique est :



    Tout changement de variable menant à une autre expression ne revient qu'à travailler sur la même variété mais avec un système de coordonnées différent. Tout ce qui se démontre dans un système de coordonnée est valable dans les autres. On pourra donc trouver tout un tas d'expression différentes de la métrique mais elles décrivent toute la même variété (ou du moins des ouverts de la même variété), la même physique. C'est quelque chose qui a été difficile à comprendre historiquement -il n'y a qu'à lire Painlevé pour s'en rendre compte, il croyait que sa solution, la métrique de Painlevé, était différente de celle de Schwarzschild- mais ça ne fait plus aucun débat aujourd'hui.

    Toute prétention selon laquelle il y aurait plusieurs solutions différentes est hors-charte car contraire au consensus scientifique.

    La signature de la métrique (+,-,-,-,) est la même qui décrit l’espace de Minkovski
    vu que c'est censé décrire notre espace-temps, ou du moins l'approximer, encore heureux que la signature est +--- ou -+++ : c'est à cela qu'on reconnait que la variété décrite possède 3 dimensions d'espace et une de temps. Toutes les solutions de la relativité générale qui décrivent ou approximent une situation physique réelle sont de signature +--- ou -+++.

    et sa revendication d’une symétrie sphérique, au contraire de la symétrie centrale revendiquée par Hilbert.
    probable insuffisance de traduction, ou bourde, parce que les expressions mathématiques dans le papier de Hilbert relèvent de la symétrie sphérique, pas de la symétrie centrale : par exemple dans l'expression B.42, les coefficients hors métrique de la sphère ne dépendent pas des angles, c'est de la symétrie sphérique, par définition, peu importe ce qui est écrit avant, il applique une symétrie sphérique pour son développement. Pour une symétrie centrale, les coefficients devraient être des fonctions périodiques des angles

    un changement de variables astucieux permet de valider son interprétation topologique, en annulant toutes les singularités, celle du centre par l’absence de l’ET, et celle à r =Rs, apparaissant comme une singularité de coordonnées qui peut être surmontée par ce changement de variables, rendant la métrique parfaitement régulière.
    comme dit plus haut, l'expression obtenue ne fait que décrire un certain morceau de la même variété, ou plutôt deux morceaux collés par une sphère...

    D’où la continuité des géodésiques de cet ET sans centre, et leur impossibilité d’entrer dans le rayon de Schwarzschild, vide d’espace, ce qui donne un démenti mathématique formel à l’argument de mach3 selon lequel l’ouvert décrivant la solution de Schwarschild ne peut être la solution complète car « certaines » géodésiques pourraient être prolongées.
    un démenti mathématique formel ? l'expression de la métrique dans les coordonnées de Painlevé-Painlevé, ou d'Eddington-Finkelstein, ou de Novikov, ou de Kruskal-Szekeres (je continue?) sont au contraire des preuves mathématiques formelles que ces géodésiques sont prolongées. Ce n'est pas en exhibant une projection Mercator de la Terre qu'on peut affirmer qu'elle ne possède pas de pôles. Ce n'est pas en exhibant une projection de Poster qu'on peut affirmer que la Terre est plate et que l'antartique est un mur infranchissable. Ce n'est pas en exhibant une projection orthographique de la terre qu'on peut affirmer que la Terre ne possède qu'un hémisphère.
    Par ailleurs on vous a démontré mathématiquement que vous aviez tord sur le sujet il y a deux ans et demi. Je me suis rafraichi la mémoire et c'est assez édifiant : https://forums.futura-sciences.com/a...trou-noir.html

    Ici aussi, maintenir de telles affirmation à contre-courant du consensus scientifique est hors-charte.

    On trouve ce changement de coordonnées sur Futura, sur le fil « Unification mécanique quantique relativité générale, (dont je suis désolé que le titre n’ait rien à voir avec le sujet), posts 9 et 20.
    il serait souhaitable d'apprendre à un insérer des liens...

    Tout s’explique clairement, sa description mathématique, sa visualisation spatiale et sa continuité revendiquées par son auteur, et l’existence d’un système de coordonnées qui permettent de « visualiser » sa topologie tout en réfutant l’argumentation factuelle la plus forte qui lui est opposée
    comme il a été répondu à l'époque :
    Citation Envoyé par Amanuensis Voir le message
    Si ça vous arrange...
    Bref, ces billevesées là, vous allez aller les raconter ailleurs maintenant, sinon il y aura un message en vert et il pourrait bien être assorti d'une sanction.

    Je suis encore d’accord avec mach3, quand il affirme que le choix de variables de départ joue un rôle.

    En effet, bien qu’elles paraissent équivalentes, et revendiquées choisies en référence à Schwarzschild, les coordonnées posées par Hilbert (ω1, ω2, ω3, ω4 =l), paraissent certes correspondre respectivement avec x, z, y et t., mais avec la différence essentielle que Hilbert pose l = it, la coordonnée temporelle serait donc devenue imaginaire.
    mais non, la coordonnée temporelle c'est t et elle est réelle. l est imaginaire et c'est une coordonnée construite à partir de t, mais ce n'est plus une coordonnée temporelle. Cette pirouette permet juste de réécrire la métrique avec un signe + devant le terme temporel comme devant les termes spatiaux, c'est un jeu d'écriture, une mode de l'époque. Suivant cette logique, la métrique de Minkowski peut s'écrire :
    (avec l imaginaire pur). En relativité restreinte, il y a quelques intérêts à faire ça, par exemple on peut traiter les rapidités comme des angles imaginaires, avec la correspondance entre cosinus hyperbolique d'un réel et cosinus d'un imaginaire. Mais bon, ce ne sont que des astuces de calcul, les résultats sont les mêmes au final.
    Je me permets de préciser que ça marche comme ça pour TOUTES les métriques possibles et imaginables en 3+1 pour décrire ou approximer une situation réelle dans notre monde : soit on écrit trois termes spatiaux de même signe et un terme temporel de signe opposé, soit on écrit 4 termes de même signe mais l'une des coordonnées est imaginaire. C'est mathématiquement équivalent, mais la 2e option, populaire du temps de Hilbert est tombée en désuétude.

    Nous nous trouvons alors dans un système de coordonnées hybride (réel/imaginaire), dans lequel les marqueurs d’espace sont réels, et celui de temps imaginaire. Une interrogation assez naturelle est alors de se demander s’il est bien raisonnable de s’appuyer sur une chimère pour décrire le réel.
    quand bien même ce serait une telle chimère, ce n'est pas un argument pour savoir si c'est raisonnable ou non de décrire le réel avec. A partir du moment ou la description du réel, peu importe si c'est avec une chimère ou non, est en accord avec les observations, alors c'est raisonnable. Il suffit de regarder les publications des expérimentateurs, à chaque fois qu'ils essaient de mettre la relativité générale en défaut, ça rate, les observations sont conformes aux prédictions, même si elles ont été faites en utilisant une chimère. La relativité générale, ça marche, jusqu'à preuve du contraire.

    D’où la réflexion que c’est précisément cette chimère qui entraîne naturellement d’avoir à considérer le, chimérique pour moi, interchange du temps et de l’espace, celui revendiqué par Aurélien Barrau dans sa conférence, simplement pour pouvoir faire exister dans ce référentiel baroque, l’impossible coexistence d’un temps propre réel avec des trajectoires réelles à la fois à l’extérieur et à l’intérieur du rayon de Schwarzschild
    Faut arrêter avec cette histoire d'inversion. Aurélien Barrau aurait pu éviter de dire ça, c'est limite irresponsable et peu excusable vu son niveau (ou alors il n'a pas le niveau qu'il prétend...). On vous a déjà expliqué, il y a deux ans et demi.

    Il se trouve que l'expression :



    correspond à de l'espace-temps vide, de symétrie sphérique pour toute valeur positive de sauf . La symétrie sphérique s'identifie par la présence de la métrique de la sphère et de l'absence des variables angulaire dans les autres coefficients. Pour le vide il "suffit" de calculer les symboles de Cristofell, puis le tenseur de Riemann, puis le tenseur d'Einstein et on trouve 0 (tenseur énergie-impulsion nul, c'est le vide), pour toutes valeurs de , pour toute valeur positive de (sauf où cela n'est pas défini), pour toute valeurs d'angle pour et .
    et ont des propriétés générales intéressantes : on voit que la métrique est invariante par translation suivant et que est un rayon aréal, c'est à dire que les ensembles d'évènements de même et de même sont des sphères de surface . Ces sphères sont concentriques suivant et forment des cylindres sphèriques suivant .

    Cette expression décrit (au moins) deux régions différentes et disjointes, régions dans lesquels et , malgré leurs propriétés générales qui ne dépendent pas de la région, vont avoir un rôle très différent.
    Quand est supérieur à , le terme en est de même signe que la métrique de la sphère : est de genre espace, par contre de terme en est de signe opposé : est de genre temps. Cela signifie, concrêtement, qu'il existe des mouvements (=des lignes d'univers) pour lesquels est constant tandis que évolue forcément de façon monotone, et les propriétés de l'espace-temps (la courbure par exemple) ne varie pas au cours de ces mouvements (aucun coefficient de la métrique ne dépend de ). Il est pertinent de reconnaitre en une variable radiale et de la renommer , et de reconnaitre en une variable temporelle et de la renommer , ce qui donne l'expression usuelle. On constate que l'expression de la métrique est invariante suivant les angles (symétrie sphérique) et suivant (stationnarité) : cela tombe bien, c'est ce qu'on cherchait initialement : une solution statique et de symétrie sphérique du vide. Dans les premières résolutions, comme celle de Schwarzschild, on imagine (ou ...) de genre espace et de genre temps d'emblée et on impose que l'expression de la métrique soit invariante suivant les angles et suivant (invariance par rotation autour d'un centre et translation temporelle). Pas de bol, si est inférieur à un certain paramètre, et n'ont plus les propriétés qu'on désire pour elles. Il aurait alors été pertinent de pas utiliser les notations et dans ce cas là...
    En effet, quand est inférieur à , le terme en est de signe contraire à la métrique de la sphère et le terme en est de même signe que la métrique de la sphère. est de genre espace et de genre temps. Il semble alors très malvenu de renommer en qui évoque du spatial alors que est temporel dans cette région, de même pour , le renommer n'est pas très malin car évoque du temporel et est spatiale. Dans cette région là, il n'existe pas de mouvement où ne varie pas de façon monotone, par contre peut rester constant. Pire, les propriétés de l'espace-temps changent au cours de ces mouvements, la courbure divergeant quand devient petit. Cette région là ne répond pas à ce qu'on cherchait au départ : l'espace-temps n'est pas statique, les coefficients de la métrique dépendent de et est temporel. Mais bon, on ne pouvait pas deviner qu'on tomberait sur deux régions avec des comportements si différents. On est parti directement avec et comme notations et comme on ne savait alors pas trop quoi faire avec cette région non statique, c'est resté comme ça. Et on s'est rendu compte que c'était bien deux morceaux de la même variété, et qu'on pouvait trouver d'autres morceaux qui les couvrent tous les deux, mais trop tard, le mal est fait, les (mauvaises) habitudes sont prises, est noté dans la région dynamique alors qu'il n'est pas spatial et y est noté alors qu'il n'est pas temporel. Utiliser des variables dont le nom n'est pas connoté, comme et , qu'on renomme ensuite en fonction du contexte aurait été plus malin, mais Schwarzschild ne pouvait pas le deviner.

    m@ch3
    Never feed the troll after midnight!

  9. #39
    bernarddo

    Re : Jamais rien ne tomba dans un trou noir ?

    Citation Envoyé par mach3 Voir le message
    mais l'emporter sur quoi ? Il n'y a qu'une seule et unique solution pour un espace-temps de symétrie sphérique, vide et asymptotiquement plat, une variété qui présente une géométrie dite "de Schwarzschild". On peut construire une infinité de système de coordonnées sur cette variété et il y a un système de coordonnées particulier, dit "de Schwarzschild", où l'expression de la métrique est :



    m@ch3
    Merci d'avoir réagi, en précisant de façon "non métaphorique" comme dirait A. Barrau, votre vision de l'unicité de la métrique de Schwarzschild, ce qui me permettra de poser une (dernière?) question encore plus triviale sur quelque chose que je comprends toujours pas, et qui n'est de plus qu'une question de vocabulaire.

    comment peut-on évoquer l'existence d'un ET vide à propos de la solution qui est établie précisément en postulant la présence à l'origine d'une masse ponctuelle M ?

  10. #40
    mach3
    Modérateur

    Re : Jamais rien ne tomba dans un trou noir ?

    Cela a déjà été expliqué il y a 2 ans et demi, dans le fil que je cite dans mon message précédent il me semble. Je crois que c'est 0577 qui donne l'explication, mais je n'ai pas le temps d'aller vérifier maintenant. Quoi qu'il en soit cela devrait faire du bien de relire ce fil, surtout les interventions d'amanuensis et de 0577.

    EDIT : c'est amanuensis et moi-même qui parlons de ce sujet dans le fil cité, pas 0577. 0577 expliquait cela dans un autre fil auquel vous avez participé que je n'ai pas encore retrouvé

    m@ch3
    Dernière modification par mach3 ; 18/07/2021 à 11h54.
    Never feed the troll after midnight!

  11. #41
    Mailou75

    Re : Jamais rien ne tomba dans un trou noir ?

    Salut,

    Désolé j'ai trainé, trop de taf en TT en ce moment...

    Citation Envoyé par bernarddo Voir le message
    Bonjour Mailou, et merci de t'intéresser à ce fil
    Pas de quoi, je m'y intéressais avant que tu arrives, j'affectionne le sujet en général

    Ensuite passons tout de suite au discours bizzaroïde d'Aurélien Barrau (...) qui l'amène à considérer qu'il y aurait une inversion de l'espace et du temps
    Sur ce point j'ai une vision pas tout à fait orthodoxe de m'expliquer (perso donc...) cette inversion. Je ne me base pas comme vous sur le signe (+ ou -) de membres d'une métrique mais plutot sur des "rotations d'aiguilles", les aiguilles représentant la direction de lignes d'univers. En gros en étant statique au bord d'un trou noir les "aiguilles" peuvent avoir un angle entre elles (observateur/observé) de la même façon qu'un angle est une vitesse en RR. L'inversion s'expliquerait alors par le fait que, pour un observateur éloigné les aiguilles (propre, espace et temps) d'un objet proche de Rs sont confondues à 45° (en représentation type Minko) et qu'au delà de Rs elles se dédoublent à nouveau mais de manière symétrique par rapport à l’extérieur. Mais bon c'est très personnel et pas assez au point pour être défendu. Je dois un schéma sur ce point pour plaider ma cause. Quoi qu'il en soit elle ne devrait rien changer au résultats.

    Maintenant, passons aux choses sérieuses (...) elle utilise en lieu et place de la coordonnée classique radiale r, la variable auxiliaire R
    Ben j'allais justement demander si tu avais étudié (réellement) les conséquences d'un tel changement de variable, R=(r3+Rs3)1/3 si j'ai bien compris... Mais j'ai l'impression que mach3 a fait le taf et qu'il arrive à la conclusion que la métrique décrite continue d'être la même. J'ai confiance en mach3, pas le genre à affirmer s'il a un doute... Par contre je me demande si ce changement de variable pourrait donner lieu à un nouveau système de coordonnées et quels seraient ses avantages ?

    dont il est facile de voir qu'elle supprime tout l'espace intérieur au rayon de Schwarzschild
    Pour ma part une formule mathématique c'est totalement opaque, il n'est pas "facile de voir" pour moi dsl...

    une hypothèse physique irréaliste, d'une masse finie, infiniment dense, et de rayon infiniment petit
    Oui... alors j'avoue qu'au fond de moi je suis plus convaincu par une matière de "densité maximale" (trou noir "plein", suite logique d'une étoile à neutrons) et donc un TN sur lequel on s'écraserait comme une m... pas d'intérieur, pas de singularité, pas d'infini, juste un maximum. Le problème étant que le trou noir se caractérise par sa compacité et pas densité, dommage... En tout cas ça ne m'empêche pas d'étudier le modèle de Schw (TN + TBlanc), en tant qu'objet mathématique amusant et spéculatif.

    c'est mathématiquement simple, pour quelqu'un comme toi qui me paraît toucher sa bille
    Détrompe toi je suis une buse en maths. Je ne fais qu'appliquer des formules que je comprends rarement, ou des années après...

    Mais il faut faire une révolution dans sa tête...
    Entamée, mais non révolue

    .....

    Je laisse mach3 te répondre sérieusement, je ne me sens pas utile ici.

    A+

    Mailou
    Trollus vulgaris

  12. #42
    mach3
    Modérateur

    Re : Jamais rien ne tomba dans un trou noir ?

    Pour résumer brièvement :

    Le tenseur de courbure contient 20 paramètres. 10 sont conditionnés par le contenu en énergie et impulsion et ils sont strictement nuls dans le vide. les 10 autres sont conditionnés par les conditions aux limites, ici la symétrie sphérique et le fait que ce soit asymptotiquement plat.

    Si on postule un espace-temps intégralement vide, la solution obtenue présente un paramètre libre (le dans le papier de Schwarzschild) et on retrouve l'espace-temps plat de Minkowski si ce paramètre est nul. A ce stade ce ne sont que des maths et à ce titre, ce n'est pas discutable, il s'agit d'une démonstration formelle : espace-temps vide + symétrie sphérique + platitude asymptotique --> géométrie de Schwarzschild complète (avec les 4 régions et ses deux singularités, l'une passée, l'autre future) avec un paramètre libre dont la géométrie de Minkowski est un cas particulier.

    Vient ensuite le théorème de Birkhoff (encore des maths, donc non discutable), qui dit que si on considère un espace-temps de symétrie sphérique avec une région centrale non vide (un astre avec une symétrie sphérique, pas nécessairement statique) et un extérieur vide, alors l'extérieur est décrit par la géométrie de Schwarzschild avec un paramètre fixé par les conditions aux limites avec la sphère centrale non vide.

    On trouve alors que ce paramètre est lié à la masse de l'astre. L'énergie-impulsion a l'intérieur de l'astre (qui peut se résumer globalement à sa masse) confère à la région centrale une géométrie telle qu'à la limite avec la région vide, le "collage" entre cette géométrie et celle de Schwarzschild est continu et doux et pour ce faire le paramètre doit être réglé finement. La géométrie de l'intérieur impose la valeur de à l'extérieur. On montre ensuite par l'étude des géodésiques situées au loin que correspond à 2M (à des facteurs G et c près), avec M la masse du corps central.

    On ne peut pas considérer un astre ponctuel, mais ça Schwarzschild ne pouvait pas le deviner. L'étude de la géométrie de Schwarzschild montre que si on considère la présence d'un astre au centre, alors le cas le plus extrême possible est une masse ponctuelle (issue de nulle part, la ligne d'univers démarrant sur la singularité passé) qui enfle en un temps fini jusqu'à ce que son rayon soit alpha puis qui s'effondre dans le même temps fini jusqu'à être de nouveau ponctuelle (la ligne d'univers termine sur la singularité future). On ne peut pas faire plus petit que ce truc qui passe d'un rayon nul à alpha puis à un rayon nul (sans passé qui précède et futur qui suit). Ca c'est encore mathématique et indiscutable. Physiquement, un tel cas ne semble pas pouvoir exister, car l'apparition de cette masse ponctuelle sur une singularité passé est sans cause (et la singularité passé est elle-même sans cause). Le cas d'un astre dont le rayon est supérieur à dans le passé et évolue jusqu'à devenir inférieur à , c'est à dire un effondrement en trou noir, est par contre beaucoup envisageable physiquement.

    L'effondrement en trou noir est un acquis au niveau mathématique, on sait produire des métriques dites d'effondrement où l'espace-temps contient un astre central dont le rayon se réduit jusqu'à 0. Au niveau physique c'est un peu plus compliqué, mais il y a des arguments et des observations assez convaincantes :
    -quand le rayon d'un corps devient inférieur à (de mémoire, à vérifier), il me semble qu'on ne connait aucun mécanisme physique (de part les propriétés de la matière dans l'astre) pour arrêter l'effondrement
    -il est exclu que l'effondrement soit stoppé juste avant que le rayon passe sous parce que la matière en effondrement passerait à un état encore inconnu sous l'effet de la pression immense, cet état s'opposant à l'effondrement : si c'était le cas, la matière dont serait faite les candidats trou noirs supermassif serait moins dense que l'eau et la pression n'y serait pas immense (du moins pas à la surface)...
    -quand on observe une étoile qui se fait avaler par un candidat trou noir, on s'attend à observer un phénomène très lumineux si le candidat possède une surface un poil au dessus de , phénomène que l'on observe pas, donc qui amène à penser qu'il n'y a pas une telle surface.

    Dans les solutions physiquement envisageables pour une symétrie sphérique, où il y a un corps central d'une certaine masse, les régions "bizarres", III et IV, l'autre univers et le trou blanc, ne sont pas présentes. Seule la région I est présente, éventuellement avec la région II si il y a effondrement avec formation d'horizon.

    Il existe bien des tentatives pour dire qu'il se passe quelque chose de particulier juste avant qu'un horizon se forme et que cela empêche cette formation (le firewall), mais ça reste encore à l'état d'hypothèse spéculative (et je ne suis pas capable d'en discuter, c'est de la théorie quantique des champs en espace-temps courbe, lié au rayonnement et à l'évaporation de Hawking).

    m@ch3
    Never feed the troll after midnight!

  13. #43
    mach3
    Modérateur

    Re : Jamais rien ne tomba dans un trou noir ?

    Citation Envoyé par Mailou75 Voir le message
    Ben j'allais justement demander si tu avais étudié (réellement) les conséquences d'un tel changement de variable, R=(r3+Rs3)1/3 si j'ai bien compris... Mais j'ai l'impression que mach3 a fait le taf et qu'il arrive à la conclusion que la métrique décrite continue d'être la même. J'ai confiance en mach3, pas le genre à affirmer s'il a un doute... Par contre je me demande si ce changement de variable pourrait donner lieu à un nouveau système de coordonnées et quels seraient ses avantages ?
    La question ne se pose même pas vu que la physique ne dépend pas du système de coordonnées qu'on utilise.

    A ce propos, je disais dans le fil d'il y a 2 ans et demi (dont je te conseille la relecture) :

    Citation Envoyé par mach3 Voir le message
    La métrique avec "r" au lieu de "R" dans le papier de Schwarzschild s'écrit, sauf erreur :



    Je ne vois pas de changement qualitatif (si ce n'est au niveau de l'esthétique ou de la commodité) par rapport à l'écriture en grand R (celle qu'on trouve aujourd'hui partout avec un petit r). r prend, a priori, une valeur allant de -alpha à +infini (de par la définition de R en fonction de r). Si r=0, on a la même singularité de coordonnées que lorsque R=rs. Si r<0, t devient de genre espace et r de genre temps, tout comme t devient de genre espace et R de genre temps quand R=rs.
    On peut être tenté d'opposer le fait que r ne pourrait pas être négatif (en commettant la bévue de se croire encore dans l'espace euclidien...), mais ce n'est pas différent de dire que R ne pourrait pas être inférieur à rs. L'argument n'a pas plus de poids, et on trouvera les mêmes arguments pour, au contraire, plaider pour une extension vers r<0 que ceux qu'on trouve pour une extension vers R<rs.
    On ne fait que décaler le "problème".
    m@ch3
    Never feed the troll after midnight!

  14. #44
    mach3
    Modérateur

    Re : Jamais rien ne tomba dans un trou noir ?

    Pour finir, je vais répondre à

    Citation Envoyé par Nickelange Voir le message
    Bonjour,

    Quelle différence entre date "non définie" et "la fin des temps" ?

    Olivier
    car le fil a été détourné et aucune réponse satisfaisante n'a été faite (enfin, j'ai pas tout relu, mais il me semble).

    désolé pour le manque de clarté. Ce que je voulais dire, c'est que depuis l'intérieur du trou noir, on ne voit pas au-dela d'une certaine date et que cette date est d'autant plus reculée que ce qu'on regarde est loin (par exemple, on voit toujours le fond diffus datant de 300000 ans après le big bang). Il existe tout un tas d'évènements postérieurs à ce qu'on verra en dernier avant la singularité. Notamment, on ne verra pas la chute de certains objets si ils tombent dans le trou noir suffisamment "longtemps" après nous (pas facile d'utiliser le vocabulaire quotidien dans ce cas...). Pour un observateur lointain qui utiliserait la coordonnée t de Schwarzschild comme datation (ce qui n'est pas malin!), toutes les traversées de l'horizon se font pour une coordonnée t arbitrairement grande, ce qui laisse penser que ces traversées ont lieu dans l'infini futur, mais cela n'est dû qu'au choix de t comme coordonnée temporelle. Si on utilise tr par exemple, la coordonnée temporelle de Gullstrand-Painlevé, les traversées d'horizon se font bien à des dates distinctes finies et on peut les ordonner. Des traversées d'horizon qui se produisent tard au sens de tr pourront ne pas être vue par un chuteur "précoce".

    m@ch3
    Never feed the troll after midnight!

  15. #45
    bernarddo

    Re : Jamais rien ne tomba dans un trou noir ?

    En effet, cela fait un bien fou de lire, et relire les mêmes documents (ceux de Schwarzschild et Hilbert dans le cas présent), en alternance avec les divers arguments postés sur les fils Futura.

    Il est toujours utile de suivre les conseils....

    C’est particulièrement vrai à propos du retour que nous offre mach3 sur le fil « Masse d’un trou noir », dont il reprend ci dessus son post 84 de l'époque, dans lequel il démontre par son calcul qu’il ne voit aucune différence entre l’écriture en r ou R de deux métriques qui n’en feraient qu’une.

    Or, une nouvelle, et plus attentive lecture du texte de Schwarzschild, me précise que R n’est pour lui qu’une quantité auxiliaire, (auxiliary quantity en anglais, Hilfsgrösse en allemand ), mais ne peut rien à voir avec un changement de coordonnées.

    Or le calcul de mach3 s’accompagne d’un changement de la coordonnée radiale r lorsqu’il lui attribue un intervalle (–α, +∞), coordonnée que Schwarzschild avait choisie réelle et positive dans son exposition du référentiel ( r = (x2+y2+z2)1/2 ), et l’amène en fait à reprendre le discours de Barrau, en introduisant la notion de genre, tout cela en refusant la translation introduite par la quantité auxiliaire, voulue explicitement par Schwarzschild : « In order that this discontinuity coincides with the origin ».

    Ce qui confirme que les deux métriques sont réellement différentes.

  16. #46
    mach3
    Modérateur

    Re : Jamais rien ne tomba dans un trou noir ?

    Or, une nouvelle, et plus attentive lecture du texte de Schwarzschild, me précise que R n’est pour lui qu’une quantité auxiliaire, (auxiliary quantity en anglais, Hilfsgrösse en allemand ), mais ne peut rien à voir avec un changement de coordonnées.
    Osef de ce que Schwarzschild pense que R est une quantité auxiliaire. De l'eau a coulé sous les ponts. Si on change r pour R ou inversement, on fait un changement de coordonnée, par définition, on passe de à ou inversement, et un changement de coordonnées ne change pas la physique vu que les coordonnées sont des champs scalaire choisis arbitrairement.

    Or le calcul de mach3 s’accompagne d’un changement de la coordonnée radiale r lorsqu’il lui attribue un intervalle (–α, +∞), coordonnée que Schwarzschild avait choisie réelle et positive dans son exposition du référentiel ( r = (x2+y2+z2)1/2 )
    Schwarzschild ne pouvait pas anticiper un truc pareil, c'est trop orthogonal aux habitudes de l'époque (mécanique classique, espace euclidien, etc). x, y et z ne sont définis pour la partie ]–α, 0[ et ce ne sont de toutes façons pas des coordonnées cartésiennes alors que c'est ce que Schwarzschild voulait que ce soit. L'ensemble des points de t constant pour x, y et z arbitrairement proche de 0 est une sphère de rayon arbitrairement proche de , ça se lit directement dans la métrique, peu importe comment elle est écrite (c'est la même chose que des point avec la coordonnée r arbitrairement proche de 0 ou avec la coordonnée R arbitrairement proche de , sauf que pour la dernière cela apparait plus clairement).

    Franchement osef d'étudier les textes de Schwarzschild et Hilbert sauf pour l'histoire des sciences. Ce qu'ils pensaient n'a aucune importance, c'est des maths. On donne une expression de la métrique et des intervalles de définition des coordonnées et ça suffit à n'importe quel bonhomme un peu dégourdi pour comprendre ce que ça décrit, peu importe ce que pensait la personne qui a abouti à cette expression. C'EST DES MATHS. ce n'est pas sujet à interprétation.

    Citation Envoyé par bernarddo Voir le message
    Ce qui confirme que les deux métriques sont réellement différentes.
    Ca c'est uniquement dans votre tête. J'ai démontré que les deux expressions étaient identiques, et je n'avais même pas besoin de le faire parce qu'il est reconnu par la communauté scientifique de manière consensuelle que ce sont les mêmes. Et ne parlons pas de marginaux qui n'entravent strictement rien à la relativité générale.

    m@ch3
    Never feed the troll after midnight!

  17. #47
    Mailou75

    Re : Jamais rien ne tomba dans un trou noir ?

    Salut,

    Citation Envoyé par mach3 Voir le message
    Pour résumer brièvement (...)
    Merci

    Citation Envoyé par mach3 Voir le message
    A ce propos, je disais dans le fil d'il y a 2 ans et demi (dont je te conseille la relecture)
    J'en ai relu des passages, pas tout... ta citation ne réponds pas vraiment à la question. La fonction R=(r3+Rs3)1/3 va modifier l'axes des r de Schw et l'axe d'espace du nouveau repère va devenir R. Ceci a forcément des implications sur les courbes tracées. La question est de savoir si le résultat est intéressant (ex: Kruskal les rayons sont à 45°, Painlevé l'axe de temps est du temps propre, etc...) ou si c'est un repère lambda ?

    Citation Envoyé par mach3 Voir le message
    J'ai démontré que les deux expressions étaient identiques, et je n'avais même pas besoin de le faire parce qu'il est reconnu par la communauté scientifique de manière consensuelle que ce sont les mêmes.
    Supposition : on est en plein quiproquo !
    En fait chaque système de coordonnées a sa propre "métrique" et celle de Schw "avec R" n'en est qu'une de plus. Par contre elles vont toutes décrire la "solution" de Schw dans le sens où les évènements décrits seront les mêmes. Avec selon le repère, une meilleure lisibilité de tel ou tel paramètre.
    Du coup Bernarddo à raison : c'est bien une autre métrique ; et il a tort : c'est la même physique qui est décrite.
    Me trompe-je ?
    Trollus vulgaris

  18. #48
    mach3
    Modérateur

    Re : Jamais rien ne tomba dans un trou noir ?

    J'en ai relu des passages, pas tout... ta citation ne réponds pas vraiment à la question. La fonction R=(r3+Rs3)1/3va modifier l'axes des r de Schw et l'axe d'espace du nouveau repère va devenir R. Ceci a forcément des implications sur les courbes tracées. La question est de savoir si le résultat est intéressant (ex: Kruskal les rayons sont à 45°, Painlevé l'axe de temps est du temps propre, etc...) ou si c'est un repère lambda ?
    Je reprécise que R est le rayon aréal qu'on note r habituellement et que r est "autre chose". En passant de R (le rayon aréal, variable radiale "habituelle") à r, ça va jouer significativement sur environ 3-4 fois rs, en effet, R et r sont égaux à moins d'un demi pourcent près pour R=4rs ou plus. La partie allant de R=rs à R~4rs va se retrouver dilatée suivant l'axe horizontal pour couvrir l'intervalle r=0 à r~4rs. Tu peux jeter un oeil à ce graphe desmos : https://www.desmos.com/calculator/l4uhtwsxhu
    Rien de bien intéressant ou utile a priori. Schwarzschild a fait l'hypothèse d'un point masse localisé en x=y=z=0, donc dans ce repère x,y,z une tranche d'espace doit tout remplir sauf ce point central. Manque de bol ce point est une sphère de rayon rs (il "suffit" de lire la métrique pour le conclure), donc la conclusion contredit l'hypothèse faite (mais il n'y avait aucun moyen de deviner qu'un point masse n'était pas possible).

    Supposition : on est en plein quiproquo !
    En fait chaque système de coordonnées a sa propre "métrique" et celle de Schw "avec R" n'en est qu'une de plus. Par contre elles vont toutes décrire la "solution" de Schw dans le sens où les évènements décrits seront les mêmes. Avec selon le repère, une meilleure lisibilité de tel ou tel paramètre.
    Du coup Bernarddo à raison : c'est bien une autre métrique ; et il a tort : c'est la même physique qui est décrite.
    Me trompe-je ?
    "chaque système de coordonnées a sa propre "métrique""... plus proprement il faut dire qu'il y a une expression différente de la même métrique pour chaque système de coordonnées. Mais là en l'occurence, c'est deux fois la même expression, dans le même système de coordonnées. Relis le post suivant : https://forums.futura-sciences.com/d...ml#post6823912
    les métriques publiées dans l'article de Schwarzschild et dans l'article de Hilbert sont identiques (à la notation près). Hilbert est juste un peu plus hardi car il s'autorise à regarder ce qui se passe pour un rayon aréal nul alors que Schwarzschild ne descend pas en-dessous de R=rs ou r=0.
    par contre au message 43 ( https://forums.futura-sciences.com/d...ml#post6825575 ), il s'agit d'une autre expression de la même métrique, dans un autre système de coordonnée avec r=(R3-Rs3)1/3, R étant le rayon aréal.

    m@ch3
    Never feed the troll after midnight!

  19. #49
    bernarddo

    Re : Jamais rien ne tomba dans un trou noir ?

    Bonjour,

    Citation Envoyé par mach3 Voir le message
    Franchement osef d'étudier les textes de Schwarzschild et Hilbert sauf pour l'histoire des sciences. Ce qu'ils pensaient n'a aucune importance, c'est des maths.
    m@ch3
    Je ne peux vous suivre sur ce point : à partir du moment où on cherche à comprendre leurs travaux, penser que ce que ces deux hommes, et en particulier Schwarzschild, avaient dans la tête est sans importance du fait que ce ne serait que des maths, me paraît un tantinet osé.

    D’abord parce que c’est inexact, qu’il s’agit d’abord de physique, et non uniquement de maths. Et ensuite, justement parce que c’est de la physique, parce qu’ils font des hypothèses de départ, qui doivent être bien comprises et prises en compte pour bien les intégrer

    1 dans des affirmations postérieures à la solution de Schwarzschild
    2 dans l’interprétation même de sa solution.

    Il se trouve que j’ai des difficultés à comprendre certaines, que je me propose de vous exposer. Il s’agit simplement de m’aider à résoudre ces difficultés

    Sur le point 1, on trouve une affirmation largement partagée par la communauté, et exprimée ici par mach3 plus haut sur ce fil

    Citation Envoyé par mach3 Voir le message
    Il n'y a qu'une seule et unique solution pour un espace-temps de symétrie sphérique, vide et asymptotiquement plat, une variété qui présente une géométrie dite "de Schwarzschild". On peut construire une infinité de système de coordonnées sur cette variété et il y a un système de coordonnées particulier, dit "de Schwarzschild", où l'expression de la métrique est :



    m@ch3
    Sachant que j’adhère entièrement à la proposition suivante et à son complément implicite, qu’à un espace-temps physiquement individualisé ne correspond qu’une seule métrique:
    Citation Envoyé par mach3 Voir le message
    "chaque système de coordonnées a sa propre "métrique""... plus proprement il faut dire qu'il y a une expression différente de la même métrique pour chaque système de coordonnées.
    m@ch3
    elle me semble recéler une impossibilité conceptuelle (quiproquo à éclaircir ?) que l’on peut exprimer de deux façons :

    a) comment peut-on associer l’idée qu’un ET en continuité avec celui dans lequel nous baignons peut être à la fois vide et non plat sur toute son étendue (puisque seulement aymptotiquement), alors que notre espace-temps vide est celui de Minkowki, qui a déjà sa propre métrique, métrique dont on revendique l’unicité ?

    b) avec son corollaire mathématique : comment un ET sans matière pourrait-il se voir muni d’une métrique affichant la constante gravitationnelle G et une masse M ?


    2 Sur le point de l’interprétation, il me semble que Schwarzschild postule, quoique seulement implicitement, un espace-temps REEL, afin qu’il puisse se raccorder asymptotiquement à l’espace réel de Minkowski. Il pose r = (x2+y2+z2)1/2 , r qui n’est pas une longueur mais un marqueur de l’espace séparant le point considéré à une origine située à r =0 dans un référentiel réel, ne peut donc pas être négatif.

    Ce point est manifestement oublié, ou non intégré, par Hilbert qui, dans son résultat, se trouve en plein accord mathématique avec mach3 quand celui-ci écrit ce qui est parfaitement vrai dans le cadre d’une physique qui ne se limiterait pas au réel, ce que les mathématiques permettent sans problème:
    Citation Envoyé par mach3 Voir le message
    Si on change r pour R ou inversement, on fait un changement de coordonnée, par définition, on passe de à ou inversement, et un changement de coordonnées ne change pas la physique vu que les coordonnées sont des champs scalaire choisis arbitrairement.
    …..
    C'EST DES MATHS. ce n'est pas sujet à interprétation.
    m@ch3
    Il me semble que le dernier mot est le plus important : arbitrairement. Hilbert, comme Mach3, n’avaient pas simplement pas le droit de faire un changement de coordonnées qui ne respecte pas l’exigence d’un r positif posée par Sch d’un ET réel, sous ensemble réel d’une physique inchangée, que les maths permettent de traiter en utilisant les nombres imaginaires, arbitraire que Hilbert assume formellement quand il pose l=it, ce qui fait à mon sens toute la différence entre les deux métriques

    L'impossibilité conceptuelle du premier point, qui assimile implicitement la topologie de l’ET de Schwarzschild avec celle de Minkowki, continue sur tout l’espace, prépare mentalement la solution venant de l’ « oubli » du second, et valide la déduction que
    « donc dans ce repère x,y,z une tranche d'espace doit tout remplir sauf ce point central »,

    ce qui ouvre donc la porte à la recherche d’une extension (ici celle de Krusk-S) de l’ET à l’intérieur de rs, ET qui, contraire à son hypothèse, n’existe pas pour Schwarzschild, ce qui fait du coup disparaître la singularité centrale.

    Toute l’astuce du calcul de Schwarzschild a été, par sa translation permettant de les faire coïncider spatialement, de neutraliser et faire disparaître un indéterminé physique (∞, si on considère la densité de la masse ponctuelle) et l'indéterminé mathématique correspondant apparu dans le calcul,∞ du même ordre, dans une division ∞/∞ permettant de revenir à des valeurs déterminées.
    Dernière modification par bernarddo ; 21/07/2021 à 10h38.

  20. #50
    mach3
    Modérateur

    Re : Jamais rien ne tomba dans un trou noir ?

    Va falloir apprendre à démêler les maths de la physique. Les variétés, les métriques, les géodésiques, les tenseurs de courbure, d'Einstein, d'énergie-impulsion, l'équation d'Einstein, tout cela n'est vraiment QUE des maths. La physique c'est quand on fait correspondre les grandeurs sorties des maths avec les grandeurs sorties des observations, soit parce qu'on utilise les observations pour construire des maths qui y correspondent, soit parce qu'on utilise les maths pour prédire des observations.

    Ici, il s'agit en premier lieu de maths. On pose une variété, on y impose que le tenseur d'Einstein est nul, ce qui est une contrainte géométrique, qu'il y est une symétrie sphérique, une autre contrainte géométrique, et enfin que le tenseur de courbure converge vers 0 asymptotiquement, encore une autre contrainte géométrique. On cherche ensuite toutes les variétés telles qu'elles respectent ces contraintes. Ces trois contraintes ont bien sûr un pendant "physique", elles veulent chacune dire quelque chose de particulier quant aux mesures que ferait un physicien vivant dans cette variété, mais cela ne rentre pas en ligne de compte dans la résolution du problème mathématique qui est de trouver toutes les variétés telles que le tenseur d'Einstein est partout nul, qu'il y a symétrie sphérique et que la courbure est asymptotiquement nulle.
    A la rigueur la physique aura son mot à dire ensuite pour éventuellement exclure des solutions, en les considérant non physique, ou au contraire trouver une situation physique où une solution peut servir, même s'il ne s'agit que d'un morceau. Par exemple il est très probable que la solution complète de Kruskal avec les 4 régions ne corresponde à aucune situation physique. Et encore on ne parle même pas du niveau d'idéalisation de cette solution, mais bon il faut bien démarrer quelque part et la physique est justement l'art de savoir faire des approximations : Supposons que les planètes et autre corps en orbite autour du Soleil sont négligeables, que les autres étoiles au loin sont négligeables, que le Soleil possède une symétrie sphérique (alors qu'en fait il tourne...) et que c'est le vide parfait autour de lui, quelles sont alors les géodésiques de l'espace-temps qui l'entoure ? On traduit ça dans les maths, on résout, on prédit les géodésiques, on en déduit les observations correspondantes et comme ça on trouve la bonne valeur pour l'avance du périhélie de Mercure. Ca c'est de la physique. Alors que trouver la solution de Schwarzschild ou démontrer le théorème de Birkhoff, les deux outils nécessaires pour cette question (encore que Birkhoff on peut s'en passer en faisant l'hypothèse que le rayon du Soleil est négligeable ), ce n'est pas de la physique, ce sont des maths.

    Une théorie physique contient ces deux éléments, un modèle purement mathématique d'un côté, et une correspondance entre certaines grandeurs du modèle et certaines grandeurs mesurables. Et il est normal que le modèle puisse aller "trop loin" par rapport à la réalité, qu'on y trouve des solutions des insensées, un bon physicien doit savoir faire le tri. Il est aussi normal qu'au début on ne sache pas bien faire la correspondance entre grandeurs, pareil, un bon physicien doit apprendre à trouver la correspondance.

    a) comment peut-on associer l’idée qu’un ET en continuité avec celui dans lequel nous baignons peut être à la fois vide et non plat sur toute son étendue (puisque seulement aymptotiquement), alors que notre espace-temps vide est celui de Minkowki, qui a déjà sa propre métrique, métrique dont on revendique l’unicité ?
    Vide en physique correspond à tenseur énergie-impulsion nul dans le modèle, alors que non plat signifie tenseur de courbure non nul dans le modèle. Le tenseur de coubure est lié au tenseur énergie impulsion au travers de l'équation d'Einstein : la nullité du tenseur énergie impulsion impose la nullité du tenseur d'Einstein qui lui-même, de part sa construction impose la nullité de certaines composantes du tenseur de courbure, mais pas toutes. Certaines restent libres et ne sont pas tenue d'être nulles si le tenseur d'Einstein est nul = si l'espace-temps est vide. On a donc plat implique vide, non vide implique courbé, mais vide n'implique rien, cela peut être courbé ou non. L'espace-temps de Minkowski, plat, est un espace-temps vide, mais un espace-temps vide n'est pas forcément l'espace-temps de Minkowski. Ca c'est des maths et c'est non discutable.
    Par contre qu'un espace-temps entièrement vide et courbé corresponde à une réalité ressort de la physique. Notre univers réel n'étant pas entièrement vide, et n'étant pas asymptotiquement plat, alors son espace-temps n'est pas décrit par cette solution. Par contre des morceaux de l'espace-temps réel vont pouvoir être approximés par des morceaux de cette solution. Un morceau dont le contenu est de symétrie sphérique, est à peu près vide et est quasiment plat sur sa frontière périphérique sera bien approximé par un morceau de la solution de Schwarzschild (qu'on aura paramétré et taillé pour qu'il s'emboite de façon lisse avec le reste de l'espace-temps, notamment avec la partie centrale du morceau qui généralement contient un astre donc est non vide et ne fait donc pas partie du morceau qu'on souhaite approximer).

    b) avec son corollaire mathématique : comment un ET sans matière pourrait-il se voir muni d’une métrique affichant la constante gravitationnelle G et une masse M ?
    Cet espace-temps n'est pas muni d'une masse. On associe le paramètre à la masse a posteriori, justement quand on doit emboiter un morceau d'espace-temps de symétrie sphérique et non vide dont le contenu représente une masse M -un astre- avec un morceau de la solution de Schwarzschild -l'extérieur vide de l'astre-, il faut fixer le paramètre libre de la solution de Schwarzschild pour que les deux morceaux correspondent et s'emboitent de façon lisse.

    2 Sur le point de l’interprétation, il me semble que Schwarzschild postule, quoique seulement implicitement, un espace-temps REEL, afin qu’il puisse se raccorder asymptotiquement à l’espace réel de Minkowski. Il pose r = (x2+y2+z2)1/2 , r qui n’est pas une longueur mais un marqueur de l’espace séparant le point considéré à une origine située à r =0 dans un référentiel réel, ne peut donc pas être négatif.
    Sauf que Schwarzschild fait des hypothèses sur la correspondance modèle/observation quand il postule une origine r=0. Il fait l'hypothèse qu'il est réellement possible d'avoir une masse ponctuelle, sauf qu'il ne sait pas à ce moment-là (vu que c'est la solution qui le dit) qu'une masse ponctuelle est interdite par le modèle mathématique. La masse qu'il pose en r=0 ne peut pas être un point (à moins d'être nulle). Dans une coupe à t=constant, le point de coordonnée r=0 ne correspond pas à un point unique mais à une sphère. Donc sans le savoir (parce qu'il ne connait pas encore la correspondance entre le modèle et les observations dans cette situation), quand Schwarzschild pose son "point masse", il postule en fait une sphère dont le rayon aréal est . Donc il développe son calcul avec l'idée que la masse est concentré en r=0 et que r=0 représente un point, sauf que la solution le contredit en montrant que la masse doit être dans une sphère de rayon aréal . Et là c'est des maths, il "suffit" de lire la métrique, l'ensemble des évènements arbitrairement proche de r=0 et de même coordonnée t est une sphère dont le rayon aréal est arbitrairement proche de .
    Du coup, fixer n'est qu'une définition inadéquate. Les champs scalaires x, y et z ne sont définis qu'à l’extérieur de la sphère et r n'est pas défini en fonction d'eux à l'intérieur. r est donc forcément positif à l'extérieur de la sphère, là où x, y et z sont définis. Et la définition de x, y et z tient du fait que Schwarzschild est parti de l'hypothèse que la masse en r=0 était ponctuelle et donc x=0,y=0,z=0 représente un point de l'espace, alors que non, x=0, y=0 et z=0 représentent une sphère de l'espace. En fait ce que fait Schwarzschild dans son article, c'est la démonstration par l'absurde qu'une masse ne peut pas être ponctuelle (c'est à dire que postuler une masse ponctuelle conduit à affirmer que cette masse n'est pas ponctuelle).

    Il me semble que le dernier mot est le plus important : arbitrairement. Hilbert, comme Mach3, n’avaient pas simplement pas le droit de faire un changement de coordonnées qui ne respecte pas l’exigence d’un r positif posée par Sch d’un ET réel, sous ensemble réel d’une physique inchangée, que les maths permettent de traiter en utilisant les nombres imaginaires, arbitraire que Hilbert assume formellement quand il pose l=it, ce qui fait à mon sens toute la différence entre les deux métriques
    Ben si, justement, c'est de plein droit. L'ensemble des points r=0 étant une sphère et les géodésiques approchant r=0 étant incomplètes il est tout naturel d'aller voir ce qui se passe pour des r négatifs qui ne signifient rien d'autre que des sphères de rayons aréals inférieurs à (toujours pareil, il suffit de lire la métrique). Quant à la remarque sur les nombres imaginaires je pense avoir déjà assez expliqué, si vous ne comprenez pas que ce n'est rien d'autre qu'une notation tant pis pour vous, au moins les lecteurs silencieux qui nous suivent péniblement auront compris eux.

    m@ch3
    Never feed the troll after midnight!

  21. #51
    bernarddo

    Re : Jamais rien ne tomba dans un trou noir ?

    Bonjour,

    Je vais essayer de reformuler ce que j’ai compris de la réponse de mach3 que je remercie d’avoir pris le temps de donner une réponse détaillée mais qui me paraît toujours irrecevable. Réponse appréciable mais compliquée à une question simple, mon questionnement sur l’incompatibilité physique, (j’insiste sur l’adjectif), qui me semble exister entre espace vide et non plat sur toute son étendue, et les conséquences que j’en tire sur le fait que les deux métriques (de Sch et de Hilbert) sont différentes.


    Première étape, l’affirmation ci-dessous. Je prends note que nous n’avons là que l’affirmation, non discutable, que les mathématiques n’interdisent pas l’existence d’une variété et donc, d’un espace-temps qui puisse être à la fois vide, plat et courbé, affirmation qui me convient et que je ne discuterai pas, n’ayant ni envie ni compétence, d'ergoter sur les subtilités des tenseurs de Ricci, de Riemann ou de Veyl, discussion d’ailleurs sans intérêt pour la suite.

    Citation Envoyé par mach3 Voir le message
    La nullité du tenseur énergie impulsion impose la nullité du tenseur d'Einstein qui lui-même, de part sa construction impose la nullité de certaines composantes du tenseur de courbure, mais pas toutes. Certaines restent libres et ne sont pas tenue d'être nulles si le tenseur d'Einstein est nul = si l'espace-temps est vide. On a donc plat implique vide, non vide implique courbé, mais vide n'implique rien, cela peut être courbé ou non.On a donc plat implique vide, non vide implique courbé, mais vide n'implique rien, cela peut être courbé ou non. L'espace-temps de Minkowski, plat, est un espace-temps vide, mais un espace-temps vide n'est pas forcément l'espace-temps de Minkowski. Ca c'est des maths et c'est non discutable.
    m@ch3
    2ème étape, inclusion dans le réel :
    Introduite directement à la suite par l’affirmation suivante :

    Citation Envoyé par mach3 Voir le message
    Par contre qu'un espace-temps entièrement vide et courbé corresponde à une réalité ressort de la physique.
    m@ch3
    Pourquoi pas, on pourrait concevoir physiquement l’existence d’espaces-temps vides, au moins au sens de masses gravitantes, et dont la courbure pourrait être tirée de l’existence de charges électriques génératrices d’énergie/impulsion.
    Mais dans notre cas, il se trouve qu’elle est expressément exclue par Hilbert:
    Nom : Capture.JPG
Affichages : 484
Taille : 33,2 Ko
    et qu’on trouverait certainement dans l’expression de la métrique des termes se rapportant à ces charges plutôt que des termes liés à la gravitation.

    J’en reste donc au fait qu’il s’agit d’une simple affirmation non démontrée, mais capitale, car affirmer l'existence réelle d'un tel ET implique qu’on doit le trouver dans notre ET, ce qui, en l’absence de justification physique par des travaux portant effectivement sur le monde réel, (observations, calculs théoriques confirmés par des expériences ?), donne à cette affirmation le caractère d’une hypothèse arbitraire.
    Il est alors assez inattendu pour moi de constater qu’un tel ET n’est pas directement utilisable puisqu’il possède des caractéristiques différentes de notre ET:

    Citation Envoyé par mach3 Voir le message
    Notre univers réel n'étant pas entièrement vide, et n'étant pas asymptotiquement plat, alors son espace-temps n'est pas décrit par cette solution.
    m@ch3
    Pour affirmer ensuite que c’est malgré tout possible, au prix d’une opération de meccano, à laquelle je trouve au moins deux bonnes raisons de ne pas accorder de crédit :
    - le fait qu’elle implique une nécessaire partition de notre ET en, au moins deux morceaux, chose qu’aucun argument appuyé sur le réel ne vient conforter et qui heurte de front l’idée de continuité qui, elle, est formellement affirmée et incluse mathématiquement dans la solution de Schwarzschild.
    - le fait qu’elle nécessite d’accepter l’idée d’une approximation, procédé si incongru ici, qu’on peut prétendre à raison qu’il fait passer la démonstration du domaine des mathématiques pures à celui des mathématiques appliquées, du calcul numérique !!

    Citation Envoyé par mach3 Voir le message
    Par contre des morceaux de l'espace-temps réel vont pouvoir être approximés par des morceaux de cette solution.
    m@ch3
    Admettons malgré tout que cette solution soit utilisable, l’affirmation suivante, qui confirme la partition de l’ET, nous offre un nouveau choc logique: on ne reconnaît plus la solution affirmée comme physiquement présente, présentée initialement comme entièrement vide et possiblement courbée, décrite maintenant comme quasiment vide et presque plate.
    Si le mot quasiment a un sens, elles sont physiquement différentes.

    Exit donc de la présence la solution initiale, sinon dans le réel, du moins dans la description !!

    Citation Envoyé par mach3 Voir le message
    Un morceau dont le contenu est de symétrie sphérique, est à peu près vide et est quasiment plat sur sa frontière périphérique sera bien approximé par un morceau de la solution de Schwarzschild
    m@ch3
    On se demande bien ce que vient faire dans cette galère la solution réellement de Schwarzschild (dont tous les composants et toutes les hypothèses sont parfaitement définis, en particulier sa masse M, de valeur quelconque, que nous écrirons MS, pour la distinguer d’un rajout de masse lié au morceau intérieur, mais sans laquelle la frontière revendiquée n’existerait pas !), appelée à la rescousse pour confirmer approximativement l’existence !! d’un morceau d’ET dont on a vu qu’il s’agissait d’une description maintenant en termes approximatifs, et différents de ceux de l’hypothèse initiale. La pure logique mathématique nous conduit à penser que c’est le seul caractère approximatif de l’hypothèse faite sur l’ET à la frontière qui impose qu’il ne puisse s’adapter qu’approximativement à la solution mathématiquement exacte de Schwarzschild.

    Seul se justifierait le recours à la métrique d’Hilbert qui admet la partition, en sus de la nécessaire fixation d'une frontière à la partition.
    Car hélas, la frontière de Schwarzschild est une frontière entre un ET (à l'extérieur), et le vide à l'intérieur de rs, alors que celle de Hilbert est une frontière entre deux morceaux d'univers emboîtés l'un dans l'autre.
    L'absence d'ET à l'intérieur du rayon, interdit à la (vraie) solution de participer à l'étape suivante d'emboîtement, étape dont elle conserve seulement la réintroduction de la masse Ms comme responsable de la courbure de l'espace à l'extérieur du rayon de Schwarzschild

    et rend superflu tout cet attirail d'hypothèses arbitraires, non justifiées, approximatives, évolutives, et nécessitant l'utilisation d'une situation littéralement oxymorique, celle que je n'ai jamais compris, le vide complet capable d'exercer un effet sur la courbure !!


    Etape finale, (pour la métrique d'Hilbert seulement), arrive l’emboîtement avec l’ET de l’intérieur du rayon, qui introduit la masse M de l’astre qu’il est censé contenir, masse forcément identique à celle de la solution de Schwarzschild, celle que nous écrivons MS, puisque c’est nécessaire pour conserver la valeur du rayon (2 GM /c2).

    Citation Envoyé par mach3 Voir le message

    (qu'on aura paramétré et taillé pour qu'il s'emboite de façon lisse avec le reste de l'espace-temps, notamment avec la partie centrale du morceau qui généralement contient un astre donc est non vide et ne fait donc pas partie du morceau qu'on souhaite approximer).
    Cet espace-temps n'est pas muni d'une masse. On associe le paramètre à la masse a posteriori, justement quand on doit emboiter un morceau d'espace-temps de symétrie sphérique et non vide dont le contenu représente une masse M -un astre- avec un morceau de la solution de Schwarzschild -l'extérieur vide de l'astre-, il faut fixer le paramètre libre de la solution de Schwarzschild pour que les deux morceaux correspondent et s'emboitent de façon lisse.
    m@ch3
    La dernière bizarrerie que l’on peut déceler dans le message, est que, lorsqu’on s’attache à établir quelque chose d’incontestable, à savoir la présence d’un ET dans tout l’intérieur de rs , (je mets de côté la singularité centrale, qui n'existe pa chez Schwarzschild car régulière et surmontée par la calcul, le minimum est de croire fermement à sa complétude. Or j’ai un énorme doute à la lecture de cette phrase :

    Citation Envoyé par mach3 Voir le message
    Par exemple il est très probable que la solution complète de Kruskal avec les 4 régions ne corresponde à aucune situation physique.
    m@ch3
    Si l’une des régions de l’espace de représentation nécessaire à expliquer l’extension de KS n’avait pas de sens physique, cela n’entraînerait-il pas de fait une nouvelle partition de l’intérieur du rayon, et ne remettrait-il pas en cause toute cette construction basée sur la séparation intérieur/extérieur du rayon ?

  22. #52
    mach3
    Modérateur

    Re : Jamais rien ne tomba dans un trou noir ?

    Je dois couper le message en deux car il contient plus de 15000 caractères...

    première partie :

    Citation Envoyé par bernarddo Voir le message
    Je vais essayer de reformuler ce que j’ai compris de la réponse de mach3 que je remercie d’avoir pris le temps de donner une réponse détaillée mais qui me paraît toujours irrecevable. Réponse appréciable mais compliquée à une question simple, mon questionnement sur l’incompatibilité physique, (j’insiste sur l’adjectif), qui me semble exister entre espace vide et non plat sur toute son étendue, et les conséquences que j’en tire sur le fait que les deux métriques (de Sch et de Hilbert) sont différentes.
    Il semblerait que j'explique très mal, car ce que j'ai dit n'a pas été compris pour l'essentiel... Je vais faire encore un effort pour être plus clair et compréhensible.
    Premièrement, comme déjà dit plusieurs fois, . Il n'y a pas de discussion possible sur le sujet. La seule différence entre les deux est le fait que Schwarzschild n'a pas "osé" aller voir ce qui se passe en , alors que Hilbert oui.

    Citation Envoyé par bernarddo Voir le message
    les mathématiques n’interdisent pas l’existence d’une variété et donc, d’un espace-temps qui puisse être à la fois vide, plat et courbé
    Ah non, pas vide, plat ET courbé, il ne peut pas être plat et courbé à la fois. C'est juste que le fait qu'il soit vide n'implique rien sur sa courbure, plus précisément sa courbure de Ricci est forcément nulle mais sa courbure de Weyl peut être non nulle. Un espace-temps peut être à la fois vide et courbé.
    On devrait même dire une région de l'espace-temps peut-être à la fois vide et courbée car cela est d'abord une loi locale : si en un évènement de l'espace-temps le tenseur énergie-impulsion est nul, alors en cet évènement le tenseur de Ricci est nul, peu importe la valeur du tenseur énergie-impulsion pour les évènements qui entoure cet évènement. C'est ce qui est écrit dans l'équation d'Einstein qui est une équation locale : l'énergie-impulsion ici et maintenant contraint la courbure de Ricci ici et maintenant, la courbure de Weyl ici et maintenant étant contrainte au contraire par ce qui se passe ni ici ni maintenant (c'est par lui que l'influence gravitationnelle se transmet à distance, sinon il n'y aurait pas de courbure à l'extérieur d'un objet pour dicter aux autres objets qu'ils doivent lui tomber dessus).
    Il en découle que si dans une région de l'espace-temps on a du vide alors que ce n'est pas vide ailleurs, alors dans toute cette région la courbure de Ricci est nulle, alors qu'elle n'est pas nulle ailleurs. Pour des raisons de continuité, il faut que la région vide possède une courbure Weyl bien particulière pour qu'à la frontière entre les régions vide et non vide la jonction soit lisse. C'est une autre façon de dire que les conditions aux limites fixent les composantes du tenseur de Riemann qui ne sont pas fixées par le contenu (=tenseur énergie-impulsion).
    Partant de cette loi locale, on peut regarder ce que cela donne si l'espace-temps est entièrement vide, en posant des contraintes géométriques (symétrie, platitude asymptotique par exemple) pour fixer le tenseur de Weyl (qui n'est pas fixé par le contenu). Mais comme je l'ai dit et il semble que j'ai été mal compris, la solution obtenue en postulant un espace-temps complétement vide ne décrit pas notre espace-temps, vu que ce dernier n'est pas vide.
    Toujours en partant de cette loi locale, on peut regarder ce que cela donne pour des régions de l'espace-temps entièrement vides, des contraintes posées par les régions non vides s'ajoutant aux contraintes géométriques éventuelles. Cette fois on peut envisager que cela décrive des régions de notre espace-temps, pour autant qu'on puisse considérer leur contenu comme négligeable et qu'elles puissent donc être considérées comme vide en bonne approximation, et ça marche, on prédit ainsi de manière suffisamment précise certaines observations (avance du périhélie de Mercure, déviation des rayons lumineux près du Soleil, effet Shapiro...). Si on veut faire mieux (c'est à dire plus proche de la réalité pour des prédictions plus fines), on peut chercher des solutions plus élaborées qui ne sont pas vides, mais la résolution analytique n'existe que de façon exceptionnelle et le supercalculateur s'impose de manière générale (il n'y a même pas de solution analytique pour le problème à 2 corps!).

    Citation Envoyé par bernarddo Voir le message
    Pourquoi pas, on pourrait concevoir physiquement l’existence d’espaces-temps vides, au moins au sens de masses gravitantes, et dont la courbure pourrait être tirée de l’existence de charges électriques génératrices d’énergie/impulsion.
    plus généralement, en application de ce qui est dit plus haut, la relativité générale prédit, physiquement, qu'une région vide est courbée notamment parce qu'il y a de l'énergie-impulsion dans les régions contigües.

    Citation Envoyé par bernarddo Voir le message
    Mais dans notre cas, il se trouve qu’elle est expressément exclue par Hilbert:
    Citation Envoyé par Hilbert, 1917
    "The singularity of this interval for vanishes only when it is assumed , i.e.: under the hypotheses (1), (2), (3) the interval of the pseudo-Euclidean geometry is the only regular interval that corresponds to a world without electricity"
    Ce passage de Hilbert ne dit pas cela. Je le traduis ici en termes "modernes" :
    "la singularité de cette expression de la métrique pour ne disparait que si on pose , c'est-à-dire que sous les hypothèses (1: coefficients non diagonaux de la métrique avec la 4e coordonnée nuls), (2: coefficients de la métrique indépendant de la 4e coordonnée) et (3: symétrie sphérique), la seule métrique qui soit sans singularité et qui corresponde avec un univers vide est celle de Minkowski".
    C'est une assertion mathématique : une variété entièrement vide et sans singularité qui respecte (1), (2) et (3) est forcément plate.
    La contraposée est qu'une variété courbe et entièrement vide qui respecte (1), (2) et (3) contient forcément une singularité. Et cela ne dit rien, a priori, sur les variétés qui ont des régions non vides et des régions vides telles que les hypothèses (1), (2) et (3) sont respectées. Rappelons que le théorème de Birkhoff, qui permet justement de dire ce qui se passe pour de telles régions n'a été démontré que 6 ans plus tard (1923).

    Citation Envoyé par bernarddo Voir le message
    J’en reste donc au fait qu’il s’agit d’une simple affirmation non démontrée, mais capitale, car affirmer l'existence réelle d'un tel ET implique qu’on doit le trouver dans notre ET, ce qui, en l’absence de justification physique par des travaux portant effectivement sur le monde réel, (observations, calculs théoriques confirmés par des expériences ?), donne à cette affirmation le caractère d’une hypothèse arbitraire.
    Comme déjà dit plus haut, on procède ainsi pour faire des prédiction de la RG dans le système solaire. On fait l'approximation que le Soleil possède une symétrie sphérique et qu'il est entouré de vide (planètes, autres corps, et rayonnements négligés), on modélise l'espace-temps de la région extérieure au soleil par la solution de Schwarzschild en choisissant le paramètre qui correspond à la masse du Soleil et on calcule les géodésiques et les observations qui doivent en découler --> avance du périhélie de Mercure, déviation des rayons lumineux, effet Shapiro, etc.

    Citation Envoyé par bernarddo Voir le message
    Il est alors assez inattendu pour moi de constater qu’un tel ET n’est pas directement utilisable puisqu’il possède des caractéristiques différentes de notre ET:
    Citation Envoyé par mach3 Voir le message
    Notre univers réel n'étant pas entièrement vide, et n'étant pas asymptotiquement plat, alors son espace-temps n'est pas décrit par cette solution.
    m@ch3
    Pour affirmer ensuite que c’est malgré tout possible, au prix d’une opération de meccano, à laquelle je trouve au moins deux bonnes raisons de ne pas accorder de crédit :
    - le fait qu’elle implique une nécessaire partition de notre ET en, au moins deux morceaux, chose qu’aucun argument appuyé sur le réel ne vient conforter et qui heurte de front l’idée de continuité qui, elle, est formellement affirmée et incluse mathématiquement dans la solution de Schwarzschild.
    Fabriquer une variété en recollant deux morceaux de variétés est tout à fait valide, de même qu'on peut découper une variété en différents morceaux, et il y a toute une litterature mathématique sur le sujet (qui vole généralement bien au-dessus de mes compétences).
    Prenez un cône tronqué et une calotte sphérique (= sphère tronquée), deux morceaux de variétés (le premier d'une variété appelée cone et le second d'une variété appelée sphère), il est possible de les coller ensemble si les rayons des troncatures sont identiques. On obtient alors une nouvelle variété continue. Certes, elle n'est pas forcément dérivable sur la jonction, ce qui implique que la courbure y diverge (singularité de courbure), mais on peut choisir les caractéristiques du cône tronqué (angle d'ouverture) pour que l'assemblage soit continu et dérivable une fois (les génératrices du cône étant alors tangentes aux grands cercles de la sphère orthogonaux à la troncature) ce qui donnera une courbure, certes discontinue à la jonction, qui ne divergera pas (*).
    Il est intéressant de transposer ce cas à un espace-temps avec une région vide (ricci nul) et une région non vide (ricci non nul) car la courbure de Ricci du cône est nulle alors que la courbure de Ricci de la sphère est positive. L'ajustement de l'angle d'ouverture du cône pour avoir une variété continue et dérivable correspond au choix de pour faire coller la métrique de Schwarzschild avec la métrique interne de l'astre massif dans cette analogie.

    A noter qu'une discontinuité dans la courbure de Ricci va se produire à chaque discontinuité de densité. Rien qu'en traversant la surface entre de l'air et de l'eau liquide, on est confrontés à un tel changement discontinu de densité et donc de courbure de Ricci.

    Citation Envoyé par bernarddo Voir le message
    - le fait qu’elle nécessite d’accepter l’idée d’une approximation, procédé si incongru ici, qu’on peut prétendre à raison qu’il fait passer la démonstration du domaine des mathématiques pures à celui des mathématiques appliquées, du calcul numérique !!
    c'est tout de même le propre de la physique que de faire des approximations... refuser les approximations confine aux mathématiques pures sans application physique possible...
    Utiliser un modèle simple pour faire des prédictions sur une situation réelle complexe qui ne correspond qu'approximativement au modèle est d'une banalité affligeante en physique : on fait ça tout le temps et on ne va pas plus loin tant que la précision est suffisante.

    Citation Envoyé par bernarddo Voir le message
    Admettons malgré tout que cette solution soit utilisable, l’affirmation suivante, qui confirme la partition de l’ET, nous offre un nouveau choc logique: on ne reconnaît plus la solution affirmée comme physiquement présente, présentée initialement comme entièrement vide et possiblement courbée, décrite maintenant comme quasiment vide et presque plate.
    Si le mot quasiment a un sens, elles sont physiquement différentes.
    Ben oui, on fait de la physique. Replaçons-nous il y a 100 ans. La relativité générale vient d'être publiée, elle consiste en des systèmes équations aux dérivées partielles non linéaires impossibles à résoudre à la main dans le cas général. Il n'y a pas de super calculateurs. Comment fait un physicien pour utiliser cette théorie alors (c'est à dire faire des prédictions avec pour les comparer aux observations) ? Il fait comme il le faisait déjà avec les théories précédentes qui présentaient déjà des insolubilités à la main.
    Deux grandes voies :
    -on approxime la théorie pour pouvoir résoudre à la main (par exemple on néglige des termes ou on linéarise les équations pour revenir à un système linéaire soluble, c'est comme ça qu'Einstein a eu les premiers indices sur le périhélie de Mercure ou la déviation des rayons lumineux, et c'est comme cela qu'on prédit les ondes gravitationnelles ou les effets Lense-Thirring et de Sitter)
    -on choisit des situations simplifiées pour lesquelles une résolution à la main est possible et on fait des approximations sur une situation réelle pour qu'elle corresponde à cette situation simplifiée pour faire des prédictions sur cette situation réelle.
    L'arrivée de l'informatique et des supercalculateurs permet aujourd'hui de faire mieux, mais ça reste toujours des approximations : les données d'entrée et les calculs ont une précision numérique et une résolution limitées, donc pareil pour les résultats.
    Never feed the troll after midnight!

  23. #53
    mach3
    Modérateur

    Re : Jamais rien ne tomba dans un trou noir ?

    seconde partie

    Citation Envoyé par bernarddo Voir le message
    On se demande bien ce que vient faire dans cette galère la solution réellement de Schwarzschild (dont tous les composants et toutes les hypothèses sont parfaitement définis, en particulier sa masse M, de valeur quelconque, que nous écrirons MS, pour la distinguer d’un rajout de masse lié au morceau intérieur, mais sans laquelle la frontière revendiquée n’existerait pas !), appelée à la rescousse pour confirmer approximativement l’existence !! d’un morceau d’ET dont on a vu qu’il s’agissait d’une description maintenant en termes approximatifs, et différents de ceux de l’hypothèse initiale. La pure logique mathématique nous conduit à penser que c’est le seul caractère approximatif de l’hypothèse faite sur l’ET à la frontière qui impose qu’il ne puisse s’adapter qu’approximativement à la solution mathématiquement exacte de Schwarzschild.

    Seul se justifierait le recours à la métrique d’Hilbert qui admet la partition, en sus de la nécessaire fixation d'une frontière à la partition.
    Car hélas, la frontière de Schwarzschild est une frontière entre un ET (à l'extérieur), et le vide à l'intérieur de rs, alors que celle de Hilbert est une frontière entre deux morceaux d'univers emboîtés l'un dans l'autre.
    L'absence d'ET à l'intérieur du rayon, interdit à la (vraie) solution de participer à l'étape suivante d'emboîtement, étape dont elle conserve seulement la réintroduction de la masse Ms comme responsable de la courbure de l'espace à l'extérieur du rayon de Schwarzschild

    et rend superflu tout cet attirail d'hypothèses arbitraires, non justifiées, approximatives, évolutives, et nécessitant l'utilisation d'une situation littéralement oxymorique, celle que je n'ai jamais compris, le vide complet capable d'exercer un effet sur la courbure !!


    Etape finale, (pour la métrique d'Hilbert seulement), arrive l’emboîtement avec l’ET de l’intérieur du rayon, qui introduit la masse M de l’astre qu’il est censé contenir, masse forcément identique à celle de la solution de Schwarzschild, celle que nous écrivons MS, puisque c’est nécessaire pour conserver la valeur du rayon (2 GM /c2).
    Je passe parce qu'à la limite de l'incompréhensible et basé sur la proposition réfutée sur laquelle on ne reviendra pas.

    Citation Envoyé par bernarddo Voir le message
    La dernière bizarrerie que l’on peut déceler dans le message, est que, lorsqu’on s’attache à établir quelque chose d’incontestable, à savoir la présence d’un ET dans tout l’intérieur de rs , (je mets de côté la singularité centrale, qui n'existe pa chez Schwarzschild car régulière et surmontée par la calcul, le minimum est de croire fermement à sa complétude. Or j’ai un énorme doute à la lecture de cette phrase :

    Citation Envoyé par mach3 Voir le message
    Par exemple il est très probable que la solution complète de Kruskal avec les 4 régions ne corresponde à aucune situation physique.
    m@ch3
    Si l’une des régions de l’espace de représentation nécessaire à expliquer l’extension de KS n’avait pas de sens physique, cela n’entraînerait-il pas de fait une nouvelle partition de l’intérieur du rayon, et ne remettrait-il pas en cause toute cette construction basée sur la séparation intérieur/extérieur du rayon ?
    La solution de Kruskal est complète, ce n'est pas une croyance, c'est des maths. Toutes les géodésiques sont complètes, elles se prolongent à la l'infini sans avoir une longueur ou durée finie (à part les géodésiques qui démarrent ou s'arrêtent sur les singularités passée et future ce qui n'est guère pire que les géodésiques d'un cône qui partent ou arrivent de son sommet...), encore mieux dans les représentations compactifiées à la Penrose elles peuvent être dessinées dans leur intégralité.

    Pour passer des maths à la physique, il est certain qu'il y a des régions de l'espace-temps réel qui ressemblent beaucoup à un morceau de la région I de la solution de Kruskal (qui est justement un morceau de ce que Schwarzschild considérait comme solution, c'est à dire avec le rayon d'un astre) simplement pour la bonne raison que nous vivons dedans !

    L'existence de régions de l'espace-temps réel ressemblant à la région II (un intérieur de trou noir derrière un horizon des évènements, la partie chez Hilbert) est consensuellement admise (c'est une conséquence de plusieurs autres faits physiques comme l'effondrement inévitable d'un astre au-dela d'une certaine compacité), mais comme elles sont par définition inobservables, cela rend la confirmation difficile (il y a cependant des observations qui peuvent trancher en la faveur de la formation d'un horizon même s'il ne peut pas être observé) et il y a de la place pour des théories alternatives (et non loufoques) qui prédisent des effets nouveaux qui empêcheraient l'apparition de telles régions.

    L'existence de régions de l'espace-temps réel ressemblant à la région I connectée à la région III par un pont Einstein Rosen est consensuellement rejetée (il y a quand même des recherches sur les trous de vers qui y sont apparentés, mais c'est théorique et spéculatif, il n'y a rien du côté observation ou expérience).

    L'existence de régions de l'espace-temps réel ressemblant à la région IV est globalement rejetée aussi (mais on a quand même cherché si on ne trouvait pas de "fontaines blanches", au cas où).

    m@ch3
    Never feed the troll after midnight!

  24. #54
    bernarddo

    Re : Jamais rien ne tomba dans un trou noir ?

    L’effort est louable, mais nul n’est tenu d’expliquer quelque chose de forcément incompréhensible pour le contradicteur basique que je suis, pour qui le langage mathématique moderne est presque de l’hébreu, mais qui a quand même le bagage et la pratique mathématique et physique des ingénieurs contemporains des protagonistes.

    Donc, inversons les rôles, voila ma vision d’une simplicité biblique des deux solutions (désolé pour le pluriel), nos désaccords seront donc certainement très faciles à repérer et à contrer le cas échéant.

    Allons donc à l’essentiel de ce qui nous sépare, et sur lequel je suis complètement d’accord sur le fond.

    Citation Envoyé par mach3 Voir le message
    Premièrement, comme déjà dit plusieurs fois, . Il n'y a pas de discussion possible sur le sujet. La seule différence entre les deux est le fait que Schwarzschild n'a pas "osé" aller voir ce qui se passe en , alors que Hilbert oui.
    Si c’est pareil, ce n’est pas différent ! la première citation est oxymorique. D’ailleurs les expressions numériques des métriques sont différentes.

    Il me reste à espérer que, à défaut d’identité vraie, les deux solutions sont compatibles, et que probablement à cause de sa « frilosité », Schwarzschild est passé à côté de l'extension, en gros, votre position.

    Les deux solutions partent apparemment des mêmes hypothèses, seul le marqueur de temps de Hilbert est un nombre imaginaire : it, ce qui semble curieux pour donner une description analytique d’un espace-temps qui nous baigne et que l’on considère, à l'instar de tous nos concepts physiques, comme réel.

    R et α figurent dans les deux solutions, R figurant comme marqueur d’espace chez les deux :

    - directement chez Hilbert,

    - par l’intermédiaire d’une translation du marqueur d’espace radial r défini par rapport à l’origine, et choisi explicitement positif, (racine de somme de carrés), quoique cette exigence ne soit pas revendiquée dans le texte de la solution, translation définie via la relation R = (r3 + α3)1/3 , qui ne figure pas dans celle de Hilbert pour qui le passage de r à R constitue un simple changement de coordonnée radiale.

    Ces initiatives de Schwarzschild, loin d'apparaître comme une manifestation de frilosité, apparaissent comme fortement motivées.
    La première, celle de choisir un r positif apparaît naturelle à l’ingénieur puisqu’il s’agit après tout de décrire un ET réel, celui qui nous entoure

    La seconde, qui le conduit à réaliser cette translation, qu'il motive expressément par la nécessité d'introduire dans le calcul le respect de la continuité de l’ET est tout aussi justifiée par le respect du réel, cette continuité à ma connaissance non contestée dans l’ET qui nous baigne, apparaît encore plus légitime dans une géométrie aussi simple que celle d’une masse ponctuelle plongée dans un espace de Minkowski.

    Schwarzschild n’aurait-il donc pas osé regarder en face ce que sa solution impliquait dans l’intervalle R < α ?

    Bien sûr que si, mais la conséquence était si évidente pour lui qu’il n’a pas éprouvé le besoin de l’expliciter : la relation devenait alors : r = (R3 - α3 ) 1/3, où r, racine d’une valeur négative, prenait donc une valeur imaginaire, ce qui était contraire à son hypothèse.

    Donc l’intervalle R < α correspondait pour lui à une absence d’ET, dont la frontière à r = α devenait un bord intérieur.

    L’espace-temps à l’intérieur du rayon α, inexistant pour lui, mais introduit par la métrique de Hilbert (et décrit ultérieurement par l’extension de Kruskal) n’aurait constitué pour lui qu’une extension dans le domaine de coordonnées imaginaires, hors hypothèse de départ, mais son décès l'a empêché de commenter cette solution.

    Rajoutons que dans le même document il revendique l'unicité de sa solution et donc sa complétude, et montre que la résolution antérieure du problème de l'avance du périhélie de Mercure par Einstein n'était qu'une solution approchée, mais numériquement très proche et impossible à départager par l'observation. C'est l'attitude d'un scientifique absolument convaincu de l'exactitude et de l'unicité et de la complétude de sa solution. Exit pour moi l'argument de la frilosité

    Qu'en est-il de la complétude. On peut ensuite évaluer le devenir des solutions sur le long terme :

    Pour Schwarzschild, l’absence d’ET quand R < α inclut évidemment l’origine r =0, il n’y a plus de singularité centrale, la solution est complète, au contraire de celle de Hilbert pour qui ce qui se passe(rait) à l’origine reste indéterminé.

    Pour lui, ce qui se passe à R = α est maintenant parfaitement connu, il s’agit d’une singularité de coordonnées, l’histoire ultérieure a pu montrer grâce à un changement de variable que les géodésiques sont continues dans sa solution sans passer la frontière.
    Il a pu ensuite apporter un complément physique à sa solution, la solution interne, qui se raccorde parfaitement à la première et conforte sa validité.L'histoire n'a pas apporté d'élément de démenti à sa solution.

    Pour Hilbert, rien de nouveau sur la singularité centrale. Et qui se passe à R =α reste bien nébuleux, on se bornera simplement à rappeler le fait que des fantômes (le rayonnement de Hawking) passent apparemment clandestinement une barrière qui est mise en sens interdit par la solution !!

    Revenons à Schwarzschild: à la lumière de ce qui précède, il faut réenvisager tout différemment la citation suivante.

    Citation Envoyé par mach3 Voir le message
    Il fait l'hypothèse qu'il est réellement possible d'avoir une masse ponctuelle, sauf qu'il ne sait pas à ce moment-là (vu que c'est la solution qui le dit) qu'une masse ponctuelle est interdite par le modèle mathématique.
    Il ne fait pas l’hypothèse qu’une masse ponctuelle est physiquement envisageable, mais la démonstration de l’exact contraire puisque sa solution démontre qu’il est impossible d’inclure une telle masse dans un ET répondant à l’équation de champ d’Einstein.
    Ce qui me semble satisfaisant pour l’esprit, en donnant une limite à des grandeurs physiques comme la densité ou la pression, ce qui me semble physiquement précieux pour envisager de façon réaliste ce qui se passe au niveau des particules quand on se trouve en présence d'astres en effondrement.

    Nous n’entrerons pas ici dans l’examen de la solution interne qui précise encore les choses, y compris la possibilité de composer des morceaux d’ET, qui respectent à la jonction des valeurs égales du "périmètre" et de la courbure.

    Le fait qu’il n’ait pas été suivi après sa mort n’empêche pas que pour moi, sa solution complète, fortement implantée dans le réel, donne des réponses simples aux multiples interrogations que pose celle qui l’a supplantée.

  25. #55
    Avatar10

    Re : Jamais rien ne tomba dans un trou noir ?

    Cela tournait en rond, et ça continue...
    Juste quelques points pour montrer cela:

    Citation Envoyé par bernarddo Voir le message
    Ces initiatives de Schwarzschild, loin d'apparaître comme une manifestation de frilosité, apparaissent comme fortement motivées.
    La première, celle de choisir un r positif apparaît naturelle à l’ingénieur puisqu’il s’agit après tout de décrire un ET réel, celui qui nous entoure
    Non, pas r mais R qui est physique.




    Il ne fait pas l’hypothèse qu’une masse ponctuelle est physiquement envisageable, mais la démonstration de l’exact contraire puisque sa solution démontre qu’il est impossible d’inclure une telle masse dans un ET répondant à l’équation de champ d’Einstein.
    ??La forme "classique" implique une masse ponctuelle "logée à l'infini" et quand on étudie ça on y voit les deux discontinuité (singularité "vraie" et de coordonnées). Et la 1ere solution de Schwarzschild était formulée différemment, celle connue est celle formulée dans des coordonnées de Droste).

    Le fait qu’il n’ait pas été suivi après sa mort n’empêche pas que pour moi, sa solution complète, fortement implantée dans le réel, donne des réponses simples aux multiples interrogations que pose celle qui l’a supplantée.
    Fortement implantée dans le réel...faut s'entendre sur les mots dans ce cas, depuis Kerr a fait du boulot pour le "réel".

    Contrairement à mach3 dont l'effort est louable (et ça lui fait un bon exercice pédagogique) je ne suis pas patient, et en tant que lecteur je me demande si après des explications répétées, faudrait pas fermer...? à force on risque de perdre le lecteur intéressé...

    Bernarddo, je suis partagé quand à votre attitude (et c'est pour cela que je suis pas patient...), vous semblez intéressé, mais borné aussi, vous semblez plus à chercher des points de contre-argumentation (qui sont "n'importe quoi") que de vraiment vouloir comprendre.
    Vous auriez dû poser plusieurs questions à mach3 suite à ses explications, et au lieu de cela vous inversez les rôles pour exposez votre vision des choses (que vous aviez déjà exposées!), et en tant qu'ancien ingénieur vous devriez savoir que ce n'est pas la forme efficace d'un bon apprentissage...troll ou pas troll, c'est à se demander...ce n'est qu'un avis bien sûr, et peut être à coté de la plaque, en tout cas je me désabonne de cette discussion, ce qui pouvait y a voir d'intéressant ayant déjà été dit et redit....
    Bonne continuation.
    Dernière modification par Avatar10 ; 09/08/2021 à 10h20.

  26. #56
    bernarddo

    Re : Jamais rien ne tomba dans un trou noir ?

    Merci en tout cas d’avoir répondu,

    Je confirme que je suis intéressé, et que c’est pour cette raison, que j’ai l’exigence de compréhension de l’ingénieur qui s’engage, et que, même si je suis incapable de reconstruire tout le détail du formalisme mathématique, je suis en mesure d’interroger toute la logique physique des hypothèses, et celle du développement mathématique jusqu’à ce qu’elles m’apparaissent inattaquables.

    …et ce serait grand dommage de se désabonner du fil après m’y avoir apporté (et à tous ceux qui n’ont pas une connaissance précise de l’établissement de la formule que vous qualifiez de « classique »), le chaînon logique manquant qui justifie le raisonnement mathématique donné par mach3, ce qui me permet une lecture complètement différente de son #52 (le § « Fabriquer… en recollant… » en particulier), que je n’avais pas imaginé comme aussi solidement relié à la solution « classique », mais comme un développement théorique qui me conviendrait d’ailleurs parfaitement, expliquant le terme de « métrique interne de l’astre massif » qui me semble faire référence à la solution interne de Schwarzschild d’un astre de densité constante, (morceau calotte sphérique de développement précédent ?,auquel cas il me semble que cette caractéristique devrait apparaître dans le rayon solution), recollé avec un cône (solution de Schwarzschild, ET quasi plat, le quasi que je conteste)

    Donc cet apport:
    Citation Envoyé par Avatar10 Voir le message
    ??La forme "classique" implique une masse ponctuelle "logée à l'infini" et quand on étudie ça on y voit les deux discontinuité (singularité "vraie" et de coordonnées)
    Ainsi, la solution « classique » exigerait de recoller deux zones :
    - une zone vide, avec une masse ponctuelle logée à l’infini, dont on ne connaît d’ailleurs pas la masse ? (la solution de Schwarzschild ?)
    - une zone déterminée par l’ « astre », de masse M, dont le volume (ou la densité) ne sont pas précisés. (la solution interne de Hilbert ?)

    Tout ceci est bel et bon, mais je ne vois pas le rapport avec l’hypothèse physique de Schwarzschild (et de Hilbert qui fait la même), d’une seule masse ponctuelle de masse M que leur calcul place, pour le premier, sur l’origine du référentiel spatial, et, pour le second, qu’il envoie naviguer sur la sphère aréale dont mach3 fait état dans son # 50
    Citation Envoyé par mach3 Voir le message
    Donc sans le savoir (parce qu'il ne connait pas encore la correspondance entre le modèle et les observations dans cette situation), quand Schwarzschild pose son "point masse", il postule en fait une sphère dont le rayon aréal est
    Je signale au passage que, sur ce point, mach3 est en complet accord avec Leonard Abrams, et que c’est précisément à cause de ce point, la sphère bidimensionnelle sur laquelle se balade le point origine de Hilbert, que celui-ci rejette la solution « classique », qu’il qualifie d’Hilbert’s error.
    A retrouver sur :
    arXiv.org > gr-qc > arXiv:gr-qc/0102055
    Avec un abstract en français facilement lisible.

    Par ailleurs, et à moins d’avoir donné une masse infinie à la masse logée à l’infini (dans la zone vide !), on ne voit pas bien pourquoi l’ET à cette origine (ou sur le rayon aréal d’ailleurs), ne serait pas asymptotiquement plat et donc mathématiquement de Minkowski. Ce qui ferait de cet ajout une opération blanche !

    Et laisse le mystère entier, qui a commis l’erreur ?

    Petit commentaire sur la métrique de Kerr. Certes, son boulot est une avancée, simplement parce que son hypothèse de départ est plus complète, dans la mesure où son astre est en rotation. Quand on supprime dans sa métrique tout ce qui correspond à cette rotation on retombe sur celle de Schwarzschild.

  27. #57
    Avatar10

    Re : Jamais rien ne tomba dans un trou noir ?

    Mince, j'ai pas cliqué sur "se désabonner" mais je le fais après ce post, inutile de me répondre...bon, il y a bcp à reprendre...je vous conseille vivement de prendre un bon cours de RG, cela vous fera déjà vous exprimez correctement car là, il faut traduire, interpréter, et c'est (peut-être...sûrement!) pour cela que ça tourne en rond (en plus de vos méconnaissances).

    Un exemple:

    Citation Envoyé par bernarddo Voir le message

    Par ailleurs, et à moins d’avoir donné une masse infinie à la masse logée à l’infini (dans la zone vide !), on ne voit pas bien pourquoi l’ET à cette origine (ou sur le rayon aréal d’ailleurs), ne serait pas asymptotiquement plat et donc mathématiquement de Minkowski. Ce qui ferait de cet ajout une opération blanche !
    .
    Ce que j'ai souligné...j'interprète le "mathématiquement" comme métrique de M, et donc si métrique de M c'est asymptotiquement plat (donc on travaille avec les outils mathématiques correspondants).
    Si j'ai bien traduis, il y a un truc quand même, grossièrement le principe d'équivalence dit de façon mathématique qu'en tout point d'une la variété, il existe un espace tangent qui est l'espace de Minkowski, mais comme on est en RG il y a courbure (même pour un truc qui serait seul dans l'ET), et celle-ci est liée aux dérivées secondes de la métrique, donc on se retrouve avec une métrique de M + dérivées premières nulles+ dérivées secondes non nulles (qui sont liées au tenseur de courbure), ça veut dire qu'on peut avoir un espace tangent de M, mais l'ET n'est pas décrit par cet espace tangent (qui ne vaut que pour un point ponctuel et infinitésimal, plus ou moins massique) mais par une autre métriqueje vous laisse trouver.... Et cela peu importe la masse logée à l'infini, la logique est comme ça, à vous lire vous mélangez les concepts. et j'espère que vous ne m'en voudrez pas de cette explication qui manque de rigueur que vous m'avez reproché avant, mais vraiment pas le temps de tout bien cadré et mach3 recadrera ce qu'il est nécessaire si il le veut.
    Je ne commente pas le reste, faudrait traduire, pas le temps.
    Cette fois je me désabonne et vous souhaite (vraiment et en toute sympathie) bon courage, car vous restez dans l'ornière, ouvrez un vrai cours cela vous sera très bénéfique.
    Dernière modification par Avatar10 ; 14/08/2021 à 01h19.

  28. #58
    Mailou75

    Re : Jamais rien ne tomba dans un trou noir ?

    Citation Envoyé par bernarddo Voir le message
    Ce qui me semble satisfaisant pour l’esprit, en donnant une limite à des grandeurs physiques comme la densité ou la pression, ce qui me semble physiquement précieux pour envisager de façon réaliste ce qui se passe au niveau des particules quand on se trouve en présence d'astres en effondrement.
    Pour une fois je suis d’accord avec toi.

    D’autant que si Schwarzschild et juste alors la solution complète de Kruskal l’est aussi, justifiant la présence du premier. Et des trous blancs on en voit pas.
    Trollus vulgaris

  29. #59
    Pio2001

    Re : Jamais rien ne tomba dans un trou noir ?

    Le trou blanc... l'inverse temporel du trou noir ?

    La physique ne manque pourtant pas de phénomènes qui sont décrits par des solutions mathématiques dans les deux sens, mais que l'on observe en réalité dans un seul sens.

    L'effondrement d'un coeur d'étoile en étoile à neutron, par exemple : on n'observe jamais d'étoile à neutrons redevenir spontanément une étoile normale en activité. Or cela n'invalide pas pour autant la notion d'étoile à neutron. Ce n'est pas parce que la solution mathématique "étoile à neutron qui redevient étoile" est est tout aussi valide que son inverse, qu'on en observe forcément partout.
    Dans un espace vectoriel discret, les boules fermées sont ouvertes.

  30. #60
    Mailou75

    Re : Jamais rien ne tomba dans un trou noir ?

    Citation Envoyé par Pio2001 Voir le message
    Le trou blanc... l'inverse temporel du trou noir ?
    Non, le passé. Le trou blanc (région IV de Kruskal) est le passé de l’extérieur (I ou III). L’extérieur est lui même le passé du trou noir (région II). Si quelque chose doit influer sur l’extérieur, c’est le trou blanc, pas le trou noir, par soucis de causalité et de sens du temps. Si le modèle est juste, on devrait voir des trous blancs.
    Trollus vulgaris

Page 2 sur 5 PremièrePremière 2 DernièreDernière

Discussions similaires

  1. Réponses: 21
    Dernier message: 26/12/2020, 20h03
  2. Mouvement dans un trou noir
    Par Nickelange dans le forum Discussions libres
    Réponses: 39
    Dernier message: 24/05/2020, 13h40
  3. Un trou noir dans le voisinage
    Par Fustigator dans le forum Actualités
    Réponses: 1
    Dernier message: 07/05/2020, 12h04
  4. Compression dans un trou noir
    Par luculus07 dans le forum Questions de base et pédagogie
    Réponses: 6
    Dernier message: 02/02/2020, 16h14